Középiskolai Matematikai és Fizikai Lapok
Informatika rovattal
Kiadja a MATFUND Alapítvány
Már regisztráltál?
Új vendég vagy?

Fórum: GEOMETRIA

  [1]    [2]    [3]    [4]    [5]    [6]    [7]    [8]    [9]    [10]  

Szeretnél hozzászólni? Jelentkezz be.
[936] Bubóka2007-11-16 06:59:56

Egyetemi jegyzetben található, a szerkesztő általi sajátos jelölési mód (szerintem)A jobb oldalon az "a" oldal a magasság, illetve a szögfelező adatai vannak. A p- pí akar lenni, így (el lett írva) nem a oldal hanem alfa szög.

Előzmény: [935] HoA, 2007-11-15 14:19:10
[935] HoA2007-11-15 14:19:10

Én sem tudom, mennyire egyezményesek, pedig nem akarom ide írni hány éve foglalkozom szerkesztési feladatokkal. Honnan vetted ezt a jelölést és mit jelent? A baloldalakat majdnem megmagyaráztad - ha jól sejtem az a oldal, az ma magasság és a fb szögfelező adott. De mit jelentenek a jobboldalak?

Előzmény: [934] Bubóka, 2007-11-11 19:58:01
[934] Bubóka2007-11-11 19:58:01

Üdv Mindenkinek!

Segítséget szeretnék kérni a következő feladathoz.Nagyon fontos lenne!

Bizonyítsuk be, hogy az alábbi háromszögszerkesztési feladatok nem szerkeszthetők euklidészi értelemben! A harmadfokú problémáknál vizsgáljuk, hogy megoldható-e szögharmadoló eszközzel.

1. (a, ha, wb ) = ( p/2, 1, 2 )

2. (a, ha, wb ) = ( 1, 1, 1 )

Nem tudom mennyire egyezményesek ezek a jelek, a w - a szögfelezőt, h- a magasságot jelentené.

[933] sakkmath2007-11-07 17:51:31

Megoldásra ajánlom a következő feladatot.

Az R pont a K középpontú kör PQ húrjának felezéspontja. Bizonyítsuk be, hogy az ábra szerinti elrendezésben SY>RX. Elnézést, a rajz most csak ilyenre sikeredett :(

[932] farkasb2007-11-05 21:38:21

Kedves HoA!

Pontosan érted, hogy mire gondoltam! Köszönöm szépen, nagyszerű megoldás!

Előzmény: [930] HoA, 2007-11-05 10:34:54
[931] Fálesz Mihály2007-11-05 13:48:22

Egyáltalán nem fitymálni akartam a mátrixos megközelítést. A gyakorlatban is mátrixokat használnak (pl. a számítógépes grafikában).

* * *

Nézzük meg inkább a kvaterniós megoldást.

Legyen u egy egységvektor, \varphi egy szög és

 q = \cos\frac\varphi2 + \sin\frac\varphi2 \cdot u,

tehát q az a kvaternió, aminek skalár része \cos\frac\varphi2, vektor része pedig \sin\frac\varphi2 \cdot u. Nézzük meg, mit csinál a következő leképezés:

 x \mapsto q\cdot x\cdot \overline{q}.

Az x kvaterniót felírhatjuk a+bu+v alakban, ahol a,b skalárok, v pedig egy u-ra merőleges vektor. Legyen w=u×v a v elforgatottja u körül derékszöggel; némi számolás után kijön, hogy

 q \cdot a \cdot \overline{q} = a\cdot |q|^2 = a,

 q \cdot u \cdot \overline{q} = 
\left( \cos\frac\varphi2 + \sin\frac\varphi2\cdot u\right)
u \left( \cos\frac\varphi2 - \sin\frac\varphi2\cdot u\right) =

=
\left( -\sin\frac\varphi2 + \cos\frac\varphi2\cdot u\right)
\left( \cos\frac\varphi2 - \sin\frac\varphi2\cdot u\right) =
\left( \cos^2\frac\varphi2 + \sin^2\frac\varphi2\right) u 
= u

és

 q \cdot v \cdot \overline{q} = 
\left( \cos\frac\varphi2 + \sin\frac\varphi2\cdot u\right)
v \left( \cos\frac\varphi2 - \sin\frac\varphi2\cdot u\right) = \left( \cos\frac\varphi2\cdot v + \sin\frac\varphi2\cdot w\right)
\left( \cos\frac\varphi2 - \sin\frac\varphi2\cdot u\right)=

 =
\left(\cos^2\frac\varphi2-\sin^2\frac\varphi2\right)v+
2\cos\frac\varphi2\sin\frac\varphi2\cdot w =
\cos\varphi\cdot v+\sin\varphi\cdot w,

tehát

 q(a+bu+v)\overline{q} = a+bu+(\cos\varphi\cdot v+\sin\varphi\cdot w).

A skalár rész és az u-val párhuzamos vektor komponens nem változik, az u-ra merőleges vektor komponens pedig elfordul \varphi szöggel. Vagyis a művelet elforgatja a vektor részt u körül \varphi szöggel.

Előzmény: [928] BohnerGéza, 2007-11-05 01:57:56
[930] HoA2007-11-05 10:34:54

Kedves farkasb!

Remélem, értem mire gondolsz. [906] szerint legyen ezúttal \vec{AB} = \bf{b} és \vec{AC} = \bf{c}. Az ABC sík normális egységvektora \bf{n}_0 = \frac{b \times c} {| b \times c |} . Az ABC síkban AB-ra merőleges egységvektor \bf{n}_1 = \frac{n_0 \times b}{|b|} , a b irányú egységvektor \bf{n}_2 = \frac{b}{|b|} . Ekkor \vec{AB} alfa szögű elforgatottjának összetevői az ABC síkban {\bf b'} = \vec{AB'} = |b|\cdot cos \alpha \cdot {\bf n}_2 + |b| \cdot sin \alpha \cdot {\bf n}_1 . A keresett pont koordinátái az eredeti rendszerben  B' = A + \vec{AB'}. Megjegyzések: 1) Látható, hogy \alpha=0 -ra B' = B 2) Vigyázni kell az irányításra! Ha ABC körüljárása az óramutató irányával ellentétes, az \alpha szöggel történő elforgatás is az lesz, és fordítva.

Előzmény: [925] farkasb, 2007-11-04 21:31:19
[929] BohnerGéza2007-11-05 02:05:36

Elfelejtettem: A számítógépnek nem kell tudni, hogy mátrixokkal számol! Neki a 6 (térben 12) paraméter kiszámítási módját, és azok felhasználását kell megadnunk.

Előzmény: [928] BohnerGéza, 2007-11-05 01:57:56
[928] BohnerGéza2007-11-05 01:57:56

Beismerem, a kvaterniókkal nem foglalkoztam, csak a síkbeliekkel, azaz a komplex számokkal. (remélem nem tévedek)

Az előző hozzászólásomban leírtak a komplex számok segítségével történő forgatásnál is kellenek (eltolás - forgatás - visszatolás) kivonás - szorzás - visszaadás alakban. Talán szebb, de a számítógép számára ugyanannyi lépés.

A sik összes egyenes és osztóviszony tartó leképezése megadható olyan speciális mátrixszal, melynek utolsó sora 0,0,1, (utolsó oszlopa pedig az origó képe és az 1-es.) azaz hat paramétert kell meghatározni. Az (x;y) képét a mátrixot az (x;y;1) oszlopvektorral szorozva kapjuk. Ez 4 szorzást és 6 összeadást jelent. (Térben 9 szorzás és 12 összeadás.)

Előzmény: [927] Fálesz Mihály, 2007-11-04 23:07:02
[927] Fálesz Mihály2007-11-04 23:07:02

Forgatni kvaterniókkal is lehet, úgy sokkal szebb.

Előzmény: [926] BohnerGéza, 2007-11-04 22:46:22
[926] BohnerGéza2007-11-04 22:46:22

Az O-t helybenhagyó leképezések egyszerűbbek, ezért ilyen az általános módszer.

Egy-egy speciális eset lehet egyszerűbb, de mindenképpen ajánlom az általánossal való foglalkozást.

Előzmény: [925] farkasb, 2007-11-04 21:31:19
[925] farkasb2007-11-04 21:31:19

Lehet, hogy félreértettük egymást, vagy csak számomra tűnik túl bonyolultnak a megoldás, ezért a félreértés elkerülése végett feltöltök egy ábrát. Lényegében a Keresett B' pont rajta van a síkon, és az AB köríven, és mondjuk AB szakasztól 55 fokkal van elforgatva.

Előzmény: [923] BohnerGéza, 2007-11-02 04:49:24
[924] Bubóka2007-11-02 13:02:27

Tisztelt Fórumozók!

Segítségeteket kérném! A napokban hallottam a geogebráról. Valaki küldjön már nekem legyen szíves egy feladatot (ami esetleg egy nyomvonalat is tartalmaz) és annak megoldását, amit a geogebrával készített el.

Köszi!

[923] BohnerGéza2007-11-02 04:49:24

A megfelelő forgatás szögének meghatározását és mátrixát, ha szükséges, megírom.

Előzmény: [922] farkasb, 2007-11-01 10:52:36
[922] farkasb2007-11-01 10:52:36

Kedves Fórumozók!

Ismételten segítségre szorulok. Adott ABC pont xyz koordinátákkal, ami meghatároz egy síkot. A pont körül hogyan tudom a síkban elforgatni B-t egy tetszőleges szöggel?

[921] farkasb2007-10-30 18:46:58

Közben sikerült megcsinálnom... Az volt a probléma, hogy rosszúl értelmeztem a szögeket.

Előzmény: [920] farkasb, 2007-10-26 19:59:27
[920] farkasb2007-10-26 19:59:27

Lehet hogy nem jó a P pont célbeli koordinátája (10,30,15). Szóval azt ne vegyétek figyelembe. Egy kis segítő ábra, ha nem lenne világos a feladat:

Előzmény: [918] farkasb, 2007-10-25 22:59:45
[919] Cckek2007-10-25 23:02:49

Köszönöm mindkettőtöknek, HoA és Bohner Géza ezeket az érdekes hozzászolásokat, nekem is van egy "csúnya" trigonometrikus megoldásom, de ez nem vetekszik egyikötök megoldásával sem. Köszi.

[918] farkasb2007-10-25 22:59:45

Kedves Fórumozók!

Lenne egy újabb kérdésem/problémám.

Térbeli koordináta transzformációl lenne szó.

Adott két derékszögű koordináta rendszer, és egy P pont.

Az eredeti koordináta rendszer origója (0,0,0)

X tengelyén A (29.955,2,556,-39,952) //50 egységre O-tól

Y tengelyén B (10.063,47,815,10,604) //50 egységre O-tól

Z tengelyén C (38.748,-14.393,28.132) //50 egységre O-tól

P(20.653,53.039,4,877)

A cél koordináta rendszer origója szintén (0,0,0)

X tengelyén A' (50,0,0)

Y tengelyén B' (0,50,0)

Z tengelyén C' (0,0,50)

Keresett a P pont cél koordináta rendszer beli x,y,z koordinátája.

3dStudio Max porgrammal csináltam meg fordítva az egészet, és a keresett P' pont koordinátái: (10,30,15)

Próbáltam kiszámolni ezt a forgatást, de sehogy sem jött össze. Ezért kérnék némi segítséget.

Ezeket a képleteket használtam:

r11= cos(gamma)*cos(béta)

r12= cos(gamma)*sin(béta)*sin(alfa)-sin(gamma)*cos(alfa)

r13= cos(gamma)*sin(béta)*cos(alfa)+sin(gamma)*sin(alfa)

r21= sin(gamma)*cos(béta)

r22= sin(gamma)*sin(béta)*sin(alfa)+cos(gamma)*cos(alfa)

r23= sin(gamma)*sin(gamma)*sin(béta)*cos(alfa)-cos(gamma)*sin(alfa)

r31= -sin(béta)

r32= cos(béta)*sin(alfa)

r33= cos(béta)*cos(alfa)

P'x= r11* Px+ r12* Py+ r13* Pz

P'y= r21* Px+ r22* Py+ r23* Pz

P'z= r31* Px+ r32* Py+ r33* Pz

Előre is köszönöm a segítséget!

[917] HoA2007-10-25 19:19:39

A KöMaL régebbi olvasói számára ismert, hogy az ilyen feladatok megoldásához, ahol a \Delta-ek szögei 10o egész számú többszörösei, jól használható a szabályos 18-szög oldalaiból, átlóiból és körülírt K köréből álló H18 hálózat. Ha ívhossz egységnek K két szomszédos csúcs közötti ívét vesszük, n egységnyi ívhez n.10o kerületi és n.20o középponti szög tartozik.

Legyenek H18 csúcsai P1,.., P18, K középpontja O, sugara R. Húzzuk be a P1P6, P2P9 és P6P9 átlókat. Legyen A=P6,C=P9 , B pedig P1P6 és P2P9 metszéspontja. Ekkor ABC a feladatban szereplő \Delta, hiszen P1P6P9\angle=100o és P2P9P6\angle=40o.

OP6P9 R oldalú szabályos \Delta. Legyen a C középpontú R sugarú kör és a BC szakasz metszéspontja E. Ekkor BE a feladatban szereplő oldalhossz különbség. OCE egyenlőszárú \Delta csúcsszöge 60o-40o=20o, COE\angle=80o , E rajta van az OP5 sugáron. De akkor E a P2P9 átló e sugárra vett tükörképén, P1P8 -on is rajta van. A BP1E\Delta -ben BP1E\angle=P6P1P8\angle=20o, a B-nél lévő külső szög 40o, P1BE\Delta egyenlőszárú, így P1 feladatunk D pontja. ADC\angle=P6P1P9\angle=30o.

Előzmény: [910] Cckek, 2007-10-21 07:31:36
[916] Draskóczy Gergely2007-10-24 22:54:32

Valóban. Köszönöm a segítséget.

Üdvözlettel Draskóczy Gergely

[915] BohnerGéza2007-10-24 17:30:32
Előzmény: [914] Draskóczy Gergely, 2007-10-24 16:30:12
[914] Draskóczy Gergely2007-10-24 16:30:12

Munkám során merült föl az alábbi probléma:

adott A pont, K kör, t egyenes

szerkesszünk geometriai úton olyan kört (2 is van) mely átmegy A ponton, középpontja t egyenesen van, érinti a K kört

Tud ebben valaki segíteni?

Gergő

[913] BohnerGéza2007-10-24 03:37:14
Előzmény: [910] Cckek, 2007-10-21 07:31:36
[985] BohnerGéza2007-10-22 21:44:51

Tisztán szerkesztő megoldás:

Előzmény: [873] bohmajster, 2007-10-09 22:58:15
[912] BohnerGéza2007-10-22 20:56:25
Előzmény: [911] BohnerGéza, 2007-10-22 04:18:31
[911] BohnerGéza2007-10-22 04:18:31
[910] Cckek2007-10-21 07:31:36

Az ABC háromszögben mA=100o,mB=40o,mC=40o. Az AB oldalt meghosszabbítjuk BD=BC-AB-vel. Határozzuk meg az ADC szög mértékét.

[909] farkasb2007-10-16 21:09:35

Ezt a hozzászólást és levezetést is köszönöm!

[908] BohnerGéza2007-10-16 19:20:29

A [895] feladatára. (az ábrán A és B felcserélve)

Előzmény: [903] HoA, 2007-10-16 15:23:06
[907] farkasb2007-10-16 18:47:41

Köszönöm szépen!!!!

Előzmény: [906] HoA, 2007-10-16 18:08:17
[906] HoA2007-10-16 18:08:17

Legyen \vec{BA} = {\bf a} , \vec{BC} = {\bf c} Ekkor az ABC sík normális egységvektora {\bf n}_0 = \frac{{\bf a}\times {\bf c}}{|{\bf a}\times {\bf c}|} és D1,2=B\pm10.n0.

Az ABC síkban BA-ra merőleges egységvektor {\bf n}_1 = \frac{{\bf a}\times {\bf n}_0 }{|{\bf a}|} - hiszen a és n0 merőlegesek és n0 egységnyi. Így E1,2=B\pm10.n1.

Előzmény: [902] farkasb, 2007-10-16 15:13:47
[905] farkasb2007-10-16 17:07:11

Oh, de béna vagyok. Igen az ábrán A és B fel van cserélve

Előzmény: [903] HoA, 2007-10-16 15:23:06
[904] HoA2007-10-16 16:34:08

Legyen adott a paralellogramma az a, b, \alpha adatokkal. A szerkesztendő X ill. Y pontok [885] szerint. A Ka Apolloniusz-kör átmérője d=b/cos\alpha és ra=d/2 sugara szintén adottnak tekinthető. Érintse az AB-vel párhuzamos e egyenes Ka-t C2-ben. Ka minden pontjára, így C2-re is igaz, hogy az AC2B\angle felezője AB-t Y-ban metszi. Legyen ABC2\Delta körülírt K körének sugara R, középpontja O2. Tudjuk, hogy a belső szögfelező a szemközti oldal felező merőlegesét a körülírt körön metszi. Legyen C2Y és K metszéspontja Q. QC2O2\Delta egyenlőszárú, mert C2O2=QO2=R. C2QO2\angle=OC2Y\angle=45 fok, tehát QO2C2\angle=90 fok, ami azt jelenti, hogy O2 e-n van.

Innen az alábbi szerkesztés adódik: Az a hosszúságú AB szakasszal párhuzamosan, tőle ra távolságra vegyük fel az e egyenest. AB f felező merőlegesének és e-nek a metszéspontja O2. Az O2 középpontú, R=O2A sugarú K kör és e (egyik) metszéspontja C2, K és f metszéspontja ( e egyenes AB-t tartalmazó oldalán ) Q. C2Q kimetszi AB-ből Y-t.

Előzmény: [890] HoA, 2007-10-11 18:08:18
[903] HoA2007-10-16 15:23:06

És az ábrán A és B felcserélendő?

Előzmény: [902] farkasb, 2007-10-16 15:13:47
[902] farkasb2007-10-16 15:13:47

Megpróbálom magyarul :)

Adott ABC pont xyz koordinátákkal. Keresett 4 db pont. -Az első kettő (D1, D2), amelyik az BA szakaszra merőleges, B-től mért távolsága +10, -10 egység, és a BD1, BD2 merőleges az ABC síkra. A másik kettő (E1, E2)pedig ugyancsak merőleges a BA szakaszra, de az ABC síkon van, és B-től mért távolsága 10 e. Itt egy szemléltető ábra is. Előre is köszönet!

[901] HoA2007-10-16 14:19:52

Első kérdés : (ld. BohnerGézáé) Az ABC síkra merőleges vetítésre gondolsz?

Második kérdés: AZ ABC síkban fekvő, B-nél derékszögű ABC* \Delta C* csúcsát keresed, ahol BC* = 10 egység?

Előzmény: [899] farkasb, 2007-10-16 12:06:35
[900] BohnerGéza2007-10-16 14:12:04

Nem érthető (számomra) a feladat! Azt a részt, hogy egy pont legyen merőleges egy szakaszra, azt valószínűleg más sem érti.

Az elején merőleges vetítés van?

Fogalmazd meg jól a feladatot, talán tudunk segíteni.

Előzmény: [899] farkasb, 2007-10-16 12:06:35
[899] farkasb2007-10-16 12:06:35

Kedves Fórumozók!

Ilyen nehezet kérdeztem (nem hinném), vagy túl egyszerű? Megköszönném, ha valaki tudna segíteni.

[898] farkasb2007-10-14 23:16:22

Egy újabb, elvileg egyszerű kérdésem lenne. Adott A B C pont a térben. -Hogyan határozhatom meg azt a pontot(pontokat), mely az ABC síkra vetítve a vetítősugár a B ponton menne át, és B ponttól 10 egységnyire van. -továbbá szükség lenne arra a pontra (pontokra), amelyik AB szakaszra merőleges, és a ABC síkon helyezkedik el, és B pontból indul, és 10 egységnyire van tőle. Előre is köszönet!

[897] Hajba Károly2007-10-13 02:03:17

Átsiklottam azon, hogy BE az egység. Így kijött.

Előzmény: [896] Hajba Károly, 2007-10-13 01:45:28
[896] Hajba Károly2007-10-13 01:45:28

Ezzel a szerkesztéssel valami gond lehet, mert nekem nem jött ki. Én AE szakaszfelezőjébe raktam L-t, de ez lényegtelen, mert AL lesz az egység.

Előzmény: [882] BohnerGéza, 2007-10-10 23:57:14
[895] Hajba Károly2007-10-13 01:03:02

Amilyen arányban növelem a négyzet kerületét, olyan arányban nő az oldalhossza is. Így elég csak az egyik oldalhosszat vizsgálni. Megnövelem 4 méterrel, de ez egyben azt is jelenti, hogy a háromszorosára nőt. Ha valamit háromszorosára növelek, az azt jelenti, hogy még kétszer hozzáadom önmagához (1+2=3). Azaz a 4 méter az eredeti hossz kétszerese, így az eredeti hossz a 4 méter fele, azaz 2 méter.

Előzmény: [894] Emilio, 2007-10-12 23:42:25
[894] Emilio2007-10-12 23:42:25

Ha egy négyzet olkdalhosszát4m -rel növeljük,kerülete háromszorosára nő.mekkora az eredeti négyzet oldalhossza?

[893] HoA2007-10-12 20:27:30

Bocs , az utolsó mondat helyesen : "Ekkor az egész A*CB' \Delta és az ennek A*C oldalán fekvő A' pont is f -nek ezen az oldalán van,..."

Előzmény: [892] HoA, 2007-10-12 20:18:06
[892] HoA2007-10-12 20:18:06

Belátjuk, hogy ha a \Delta nem egyenlőszárú, pontosabban ha AB\neAC , akkor BC'=CB' és B'A'=C'A' egyszerre nem állhat fenn. Legyen az ABC\Delta -ben AC>AB, CB'=BC', BB' és CC' metszéspontja P, PA és BC metszéspontja A'. A Ceva tétel szerint BA'.CB'.AC'=A'C.B'A.C'B CB' = C'B -vel egyszerűsítve BA'.AC'=A'C.B'A . AC>AB miatt B'A>AC' és így A'C<BA' , vagyia A' a BC oldal A* felezőpontja és C között van.

A C'BA* és az A*CB' \Delta két oldalban megegyezik, de a közrezárt szög - mint az ABC \Delta nagyobb oldalával szemközti szög - az előbbiben (\beta) nagyobb mint az utóbbiban (\gamma) . Ezért a cosinus tétel szerint C'A*>B'A*. A* tehát a B'C' f felező merőlegesének ugyanazon az oldalán van, mint B'. Ekkor az egész A*BC' \Delta és az ennek A*C oldalán fekvő A' pont is f -nek ezen az oldalán van, vagyis B'-höz közelebb, mint C' -höz , A'B'<A'C' , amit bizonyítani akartunk.

Előzmény: [889] Gyöngyő, 2007-10-11 17:35:44
[891] Sz_Z2007-10-11 23:36:42

Itt egy "szerkesztő megoldás", igaz, használ némi projektív geometriát. A [874] hozzászólás alapján olyan egyeneseket keresünk az A és B pontokon keresztül, amelyek "száraktól mért dőlésszöge egyenlő ellentétes irányban" (1), és a metszéspontjuk CD-n van. Az (1) tulajdonságú egyenesek - mivel egymáshoz projektív, A ill. B tartójú sugársorok egymásnak megfelelő egyenesei - metszéspontjai egy hiperbolán vannak, melyre A és B illeszkedik. Ha ennek a hiperbolának három további tetszőleges pontját megszerkesztjük (K, L, M), akkor a hiperbola és a CD egyenes metszéspontjai szerkeszthetőek. (Természetesen ezt a szerkesztést - Steiner-szerkesztés - körzővel-vonalzóval picit hosszabb megcsinálni, mint géppel.)

[890] HoA2007-10-11 18:08:18

Amíg nem születik igazi szerkesztő megoldás, addig itt van egy , az eddigieknél talán egyszerűbb, számítást követő szerkesztés.

Legyen adva a paralellogramma az AB = a , BC = b oldalhosszakkal és az A-nál lévő \alpha szöggel. Vegyük fel az ABX \Delta Apollonius körét. Középpontja legyen O, Y-nal szemközti pontja Z. Az XYZ derékszögű \Delta ből YZ=d=b/cos\alpha , ami adatainkból számítható/szerkeszthető. Mivel az Apollonius kör pontjaira az A-tól és B-től mért távolságok aránya állandó, YB-t x-el jelölve felírható :

 \frac {YB}{YA} = \frac {ZB} {ZA} ; \frac {x} {a-x} = \frac {d-x} {d+a-x}

Rendezve x-re másodfokú egyenletet kapunk, melynek megoldása:  x = \frac {(d+a) \pm \sqrt{d^2 + a^2}} {2} Mivel x a-nál és d-nél is rövidebb, ezért \frac {(d+a)}{2} -nél is, tehát csak a negatív előjelet kell figyelembe venni. x szerkeszthető pl így: Vegyük fel a d hosszúságú YZ Z-n túli meghosszabbítására az a hosszúságú ZT szakaszt. Z-ben emeljünk merőlegest YT-re, forgassuk rá Z-ből ZT-t, a metszéspont legyen U. Ekkor YU = \sqrt {d^2 + a^2} , ezt Y-ból leforgatva YT-re a metszéspont legyen V. Ha W a VT szakasz felezőpontja, VW = WT = x.

Előzmény: [881] BohnerGéza, 2007-10-10 22:47:04
[889] Gyöngyő2007-10-11 17:35:44

Sziasztok!

Tudnátok segíteni a következő feladatban:

Adva van egy egy háromszög,felveszünk benne egy p pontot. Összekötjük a csúcsokat a szembelévő oldallal a p ponton keresztül.A metszéspontok rendre A', B',C'. Tudjuk továbbá,hogy C(B)'=B(C)',vmint B'A'=A'C'. Mutassuk meg h a háromszög egyenlőszárú.

Köszönettel: Zsolt

[888] titok1112007-10-11 15:47:34

Igen, ezt már tapasztaltam... Sajna nincs semmi ceruza, körző, vonalzó nálam, így csak az ábrába tudok rajzolgatni. Másik tipp: AB oldalra thalés kört rajzolok, és az AB flezőpontján átmenő, BC oldallal párhuzamos egyenesnek a metszéspontja nem esik egybe a keresett egyenessel? (mármint ami A-ból indul és X-be tart?) Vagy ez is speciálisan itt igaz?

Előzmény: [887] SmallPotato, 2007-10-11 13:45:26
[887] SmallPotato2007-10-11 13:45:26

"Úgy tűnik". Szó szerint. :-((

Semmi nem magyarázza, hogy úgy legyen. Ha elképzelsz egy "derékszögűbb" paralelogrammát, akkor az említett háromszög elannyira nem lesz egyenlőszárú, hogy a két "szár" egy derékszögű háromszög befogója ill. átfogója felé tart (miközben a másik befogó nagyon nem nulla).

Az eredeti hozzászólás ábrája eléggé spécire sikeredett, mert a paralelogramma hegyesszöge kb 60 fokos, emellé az AX' és az AD hossz is közel egyforma. De ezek tök esetleges dolgok.

Előzmény: [885] titok111, 2007-10-11 13:28:37
[886] titok1112007-10-11 13:40:23

No meg (de ez gondolom nem nagy felfedezés..) ha az AD oldalegyenest metszük a BX egyenesével (P pont), a BC oldalegyenest az AX egyenessel (Q pont) Akkor ABPQ egy trapéz, amelynek átlói X pontban metszik egymást.

Előzmény: [884] SmallPotato, 2007-10-11 13:16:58
[885] titok1112007-10-11 13:28:37

Szerintem lehet, hogy benéztem... Viszont úgy tűnik, (ha az eredeti ábrát nézzük (868 sz. hozzászólás itt)), hogy ha behúzzuk a paralelogramma középvonalát, akkor ahol metszi az xx'egyenest (legyen P pont), akkor ha a BP egyenest elmetszük az AD oldalegyenessel, egy egyenlőszárú háromszöget kapunk. Nem?

Előzmény: [884] SmallPotato, 2007-10-11 13:16:58
[884] SmallPotato2007-10-11 13:16:58

Ha jól értelmezem a leírásod, akkor ha a paralelogramma egy téglalaphoz tart, a megoldásod szerinti X pont a CD oldal C felőli negyedelőpontjához tart. Ez viszont így nem lehet jó; ha a paralelogramma derékszögű, úgy a helyes megoldás a CD oldal felezőpontja.

Rosszul értettem valamit? Próbálj egy ábrát berakni!

Előzmény: [883] titok111, 2007-10-11 12:08:48
[883] titok1112007-10-11 12:08:48

Bármelyik paralelogrammában: az alapot helyén hagyva csinálj egy téglalapot, amelynek oldala megegyzik az eredeti paralelogrammáéval. Ennek felső oldalát felezd el, majd az alsó oldalegyenest jobbra (most) hosszabbítsd meg, mérd fel rá a rövidebb oldalt.ezt és a téglalap fleső oldalát felező pontot kösd össze. Ahol metszi a paralelogramma tetjét, attól jobbra eső szakaszt felezd el, és megvan a X pont.

Előzmény: [882] BohnerGéza, 2007-10-10 23:57:14
[882] BohnerGéza2007-10-10 23:57:14
Előzmény: [881] BohnerGéza, 2007-10-10 22:47:04
[881] BohnerGéza2007-10-10 22:47:04

Egyszerűbb nem sikerült. A szerkesztést Euklides-ben végigcsináltam, jó lett.

Előzmény: [873] bohmajster, 2007-10-09 22:58:15
[880] Hajba Károly2007-10-10 20:45:29

Én meg arra jutottam, hogy az AB szakaszfelezője és az XX' egyenes éppen azon a köríven metszik egymást, amit az ABX-re lehet illeszteni. Talán valakinek ad valami löketet.

Előzmény: [873] bohmajster, 2007-10-09 22:58:15
[879] bohmajster2007-10-10 17:53:35

Esetleg átlehetne fogalmazni a feladatot: szerkesszük meg AL és BK félegyenest, úgy hogy dőlésszögük ugyanakkora legyen ellentett irányban, és metszetük a CD-n legyen rajta.

[878] SmallPotato2007-10-10 17:14:34

Avagy például (lévén két pontja és középpontjának AB egyenese ismert) megszerkeszthető az az Apolloniosz-kör, amelynek P pontjaira \frac{AP}{BP}= \frac{AX}{BX} =\frac{AX'}{BX'}.

Előzmény: [877] SmallPotato, 2007-10-10 17:01:36
[877] SmallPotato2007-10-10 17:01:36

Én odáig jutottam (lehet, hogy a zsákutcában? :-) ), hogy az ABX\Delta AB=c oldalának, fc szögfelezőjének és mc magasságának hossza ismert; ebből kellene ABX-et megszerkeszteni. A szögfelező és a magasságvonal hosszából megszerkeszthető az X'McX \Delta (ahol Mc az mc magasság talppontja), de ezután? ...

Előzmény: [873] bohmajster, 2007-10-09 22:58:15
[876] SmallPotato2007-10-10 16:42:51

DAX és CBX\angle a két bejelölt \alpha\angle váltószögei (mivel AD, X'X és BC párhuzamosak), tehát szinusztétel nélkül is láthatóan egyenlők.

Előzmény: [875] rizsesz, 2007-10-10 16:31:37
[875] rizsesz2007-10-10 16:31:37

XCB+XDA szög = 180, továbbá az ezekre a háromszögekre felírt szinusz-tételek miatt látható, hogy DAX=CBX szög.

Előzmény: [873] bohmajster, 2007-10-09 22:58:15
[874] Python2007-10-10 14:38:37

Kb. egy perc alatt erre jutottam: \frac{AX}{BX}=\frac{AX'}{BX'}=\frac{DX}{CX} (XX' szögfelező ABX-ben és AX'=DX, BX'=CX)

Ebből szerintem el lehetne indulni valahogy.

Előzmény: [873] bohmajster, 2007-10-09 22:58:15
[873] bohmajster2007-10-09 22:58:15

Üdv mindenkinek, már jó pár napja töröm magam egy feladaton, de nem tudom megoldani a problémát. Gondoltam megosztom veletek is:

Adott ABCD paralelogramma. Megkell szerkeszteni X-et a CD-n, úgy hogy érvényes legyen: AXX' szög ugyanakkora mint X'XB és X'X párhuzamos AD-vel. (az ábra szerint)

Várom a hozzászólásokat...

[872] BohnerGéza2007-10-07 20:33:17

Köszönöm SmallPotato megoldását a 124. feladatra!

A téma iránt érdeklődőknek javaslom, gondolkodjanak a közös belső érintők esetén is az analóg feladaton.

Előzmény: [870] SmallPotato, 2007-10-07 01:14:42
[871] SmallPotato2007-10-07 11:47:14

Néhány kiegészítés:

1.) Az eredeti 123. feladatban a hatványvonal a k1 és k2 érintési pontjában húzható közös érintő.

2.) Az előző hsz-ban leírt bizonyítás a rajzolt viszonyokon túl kiterjed az egymást érintő és az egymást metsző k1 és k2 körök esetére is. Ezekben az esetekben a hatványvonal rendre k1 és k2 közös belső érintője (értelemszerűen a körök egyetlen közös pontján át), ill. a körök metszéspontjain át húzható egyenes.

3.) Ha k1 és k2 nem érinti egymást, akkor (akár 0, akár 2 közös pontjuk van) az általuk "közrefogott" C1C2 átmérőjű körhöz G pontban húzható érintő is természetesen párhuzamos e-vel, e1-gyel és e2-vel. G aszerint helyezkedik el a közös átmérőegyenesnek a D felőli ill. azzal ellentétes oldalán, hogy k1 és k2 közös pontjainak száma 2 vagy 0.

Előzmény: [870] SmallPotato, 2007-10-07 01:14:42
[870] SmallPotato2007-10-07 01:14:42

Megoldás a 124. feladatra.

Rajzoljunk egy - szándékosan "elrontott" - ábrát, azaz egy olyat, ahol a találomra választott D pontból húzott DA és DB egyenesek által a k1 ill. k2 körökből kimetszett E1 és E2 pontok nem a közös érintő érintési pontjai!

Ebből az ábrából is megállapíthatjuk a következőket:

1.) Ha D a k kör pontja, akkor a jelölt e, e1 és e2 érintők párhuzamosak, hiszen k1-et A-ból, k2-t B-ből kellő arányban nagyítva a k1 ill. k2 körök k-ba, az E1 ill. E2 pontok D-be, tehát az e1 ill. e2 egyenesek e-be mennek át.

2.) A Thalész-tétel megfordítása értelmében az AC1E1, ABD és C2BE2 háromszögek derékszögűek (így hasonlóak is), és a vastagítva rajzolt DE1GE2 négyszög téglalap.

3.) A jelölt C1AE1 és C1E1e1 szögek egyenlőek, mert mindkettő a k1 kör C1E1 ívén nyugvő kerületi szög (utóbbi érintőszárú).

4.) Ezek szerint e1 és e2 akkor esik egybe, ha a jelzett DE1GE2 téglalapban pl. az E2E1D szög a jelzett szög pótszöge, azaz - a korábban említett háromszög-hasonlóságokat figyelembe véve - ha pl.  \frac {DE_2}{DE_1}= \frac{DA}{DB}.

Az utóbbi egyenlőséget átrendezve (az alsó ábrán csak a lényeg van kiemelve): DB*DE2=DA*DE1, azaz e1 és e2 akkor és csak akkor esik egybe, ha a D pont hatványa a k1 és k2 körökre azonos. Ez akkor és csak akkor teljesül, ha D pont a k1 és k2 körök hatványvonalán van. A hatványvonal ismert tulajdonsága, hogy adott pontjából a két körhöz egyenlő hosszú érintőszakaszok húzhatók (ha egyáltalán húzhatók), tehát (egyebek közt) a 124. feladat eredeti ábráján az F pontból is.

Előzmény: [857] BohnerGéza, 2007-09-20 11:08:40
[869] farkasb2007-10-05 03:01:23

Köszönöm, ez a 3. megoldás nagyon jó! Persze elismerésem a másik kettőért is :)

[867] HoA2007-10-04 17:01:42

Tovább egyszerűsíthető a megoldás, ha a koordinátarendszer kezdőpontját x0 -ba helyezzük át. Ekkor a \Delta két oldalvektora a=x1-x0=(-0,1693;-0,7763;-1,7028) és b=x2-x0=(-2,7655;-1,4626;0,8991) . A \Delta síkját e két vektor feszíti ki, így R felírható

R=x.a+y.b..........(1)

alakban. R az oldalfelező merőlegeseken van, tehát (R-0)a=(a-R)a , a(2R-a)=0 és hasonlóan b(2R-b)=0 . (1) -ből R felbontását a két utóbbi egyenletbe helyettesítve és rendezve kapjuk:

2.a2.x+2.ab.y=a2 ; 2.ab.x+2.b2.y=b2

Látható, hogy a numerikus számításhoz csak az a2,b2 és ab értékekre van szükségünk, ezek rendre 3,53083202 ; 10,59556982 ; 0,07262805 . A megoldás - például az egyenlő együtthatók módszerével

 x =   \frac { b^2  (ab - a^2 ) } {2 \cdot  ab \cdot ab  - 2 \cdot  a^2 \cdot  b^2  } ;  y = \frac { a^2  (ab - b^2 ) } {2 \cdot  ab \cdot  ab  - 2\cdot   a^2 \cdot  b^2  }

Numerikusan x = 0,489784223 ; y = 0,496642741 , amiből R = (-1,45639 ; -1,10661 ; -0,38747 ) . Visszatérve az eredeti koordináta rendszerre ezt x0 -hoz hozzáadva a várható r=(0,037714;-0,96211;0,532327) eredményt kapjuk.

Előzmény: [866] jonas, 2007-10-04 11:57:49
[866] jonas2007-10-04 11:57:49

Most nézzük ugyanerre az egyszerűbb megoldást.

Legyen tehát a háromszög három csúcsába mutató vektor

x0=(1.4941,0.1445,0.9198)

x1=(1.3248,-0.6318,-0.7830)

x2=(-1.2714,-1.3181,1.8189)

Egyszerűen kiszámoljuk két csúcs felezőmerőleges síkját, aztán egy másik két csúcs felezőmerőleges síkját is (a harmadik nem kell), majd a csúcsok síkját, végül ezek metszetét.

Az a és b pontok felezőmerőleges síkjának egyenpete  0 = ({\bf r} - \frac{{\bf a}+{\bf b}}2) ({\bf b}-{\bf a}) , átalakítva r(b-a)=(a+b)(b-a)/2.

Speciálisan a x0 és x1 csúcsok felezőmerőleges síkjának koordinátái

x1-x0=(-0.1693,-0.7763,-1.7028)

(x0+x1)(x1-x0)/2=-0.1659

Hasonlóan x1 és x2 csúcsoké

x2-x1=(-2.5962,-0.6863,2.6019)

(x1+x2)(x2-x1)/2=1.9474

A három csúcs síkját pedig egy 4 méretű homogén lineáris egyenletrendszerből kapjuk meg. Ha a sík egyenlete pr=q, akkor ebben xi rajta fekszik, így 0=pxi-q, amit meg kell oldani a p,q négy változóra. Ennek a megoldását például a Cramer-szabályból kapjuk meg: q a csúcsokból álló mátrix determinánsa, p pedig azoknak a mátrixoknak a determinánsa, amit ez előbbi mátrixból egy oszlop 1-re cserélésével kapunk.

p=(-3.1885,4.8613,-1.8992)

q=-5.8084

Most már csak el kell metszenünk a három síkot. Ehhez az alábbi egyenletrendszerünk van:


\left(\matrix{
-0.1693& -0.7763& -1.7028\cr
-2.5962& -0.6863&  2.6019\cr
-3.1885&  4.8613& -1.8992
}\right) {\bf r} = \left(\matrix{
-0.1659\cr  1.9474\cr -5.8084
}\right)

Ezt megoldva

r=(0.0377,-0.9621,0.5323)

ami szépen egyezik az előző megoldással.

[865] farkasb2007-10-03 10:04:35

Köszönöm!

[864] jonas2007-10-02 16:57:08

Nézzünk egy példát.

Vegyük azt a háromszöget, aminek a csúcsainak koordinátái

x0=(1.4941,0.1445,0.9198)

x1=(1.3248,-0.6318,-0.7830)

x2=(-1.2714,-1.3181,1.8189)

Ekkor az oldalak vektorai

v0=x1-x2=(2.5962,0.6863,-2.6019)

v1=x2-x0=(-2.7655,-1.4626,0.8991)

v2=x0-x1=(0.1693,0.7763,1.7028)

Most számoljuk ki az oldalak által bezárt szögeket az oldalvektorok skalárszorzatából.

|v1|=3.25512

|v2|=1.87908

(v1,v2)=-0.0726366

cos (\pi-\alpha0)=(v1,v2)/(|v1|.|v2|)=0.0118753

\pi-\alpha0=1.58267

\alpha0=1.55892

Hasonlóan

\alpha1=1.05618

\alpha2=0.52649

Ebből

sin (2\alpha0)=0.0237489

stb, amiből a körülírt kör középpontjába mutató vektor

 {\bf r} = \frac{\sin(2 \alpha_0){\bf x}_0 + \sin(2 \alpha_1){\bf x}_1 + \sin(2\alpha_2){\bf x}_2}{\sin(2\alpha_0) + \sin(2\alpha_1) + \sin(2\alpha_2)}  = ( 0.0377, -0.9621,  0.5323)

Ennek a távolsága valóban minden csúcstól ugyanannyi:

r=|r-x0|=|r-x1|=|r-x2|=3.49585

és a r-bfxi vektorok lineárisan összefüggők, azaz a pont a háromszög síkjában van, tehát ez valóban a háromszög középpontja.

Előzmény: [863] jonas, 2007-10-02 15:37:50
[863] jonas2007-10-02 15:37:50

Igen, noha fejből nem tudtam megmondani. Súlyozni kell a csúcsokat a csúcsban lévő szög kétszeresének a színuszával, és normálni ezeknek az összegével.

Előzmény: [861] farkasb, 2007-10-02 00:06:56
[862] Lóczi Lajos2007-10-02 13:29:35

Minden attól függ, hogy milyen matematikai szövegkörnyezetben vagyunk. Pl. a háromszög is tekinthető körnek (és viszont), ha topológiai szemmel nézzük a dolgokat, ahol is csak a folytonos egymásba deformálhatóság számít.

Előzmény: [860] Pokolfajzat, 2007-10-01 20:21:37
[861] farkasb2007-10-02 00:06:56

Tisztelt Fórumozók!

Érdeklődnék, hogy valaki tud-e képletet egy egy térbeli háromszög köré írható kör középpontjának kiszámítására, és a térbeli kör egyenletének felállítására. Előre is köszönettel: F.Balázs

[860] Pokolfajzat2007-10-01 20:21:37

Nincs különösebb jelentősége, csak részemről volt ez egy feltevés, másoknak. Akik azt mondták erre, hogy ez nem igaz, csakhogy az igazamat én sem tudtam ésszerűen alátámasztani. És reméltem, ha a feltevésem igaz akkor itt talán tudnak rá egyesek magyarázatot is adni.

[859] Lóczi Lajos2007-09-22 20:09:01

Tekinthető. De mit értesz ezalatt? Milyen előny származik ebből? Milyen tulajdonság lesz kiolvasható? Miért jó ez a társítás?

Előzmény: [858] Pokolfajzat, 2007-09-22 19:44:13
[858] Pokolfajzat2007-09-22 19:44:13

Csak egy kérdéssel zavarnám meg a tisztes társaságot. Előre is elnézést kérek, ha banális vagy ostoba lenne a kérdés. Tekinthető e a kör egy végtelen sokszögnek? Előre is köszönöm.

[857] BohnerGéza2007-09-20 11:08:40

124. feladat: az ábra alapján általánosíthatjuk a 123. feladatot. Bizonyítsuk az általánosítást! ( F az E1 E2 felezőpontja. )

Előzmény: [856] SmallPotato, 2007-09-20 08:39:23
[856] SmallPotato2007-09-20 08:39:23

Felhozom kicsit a topikot.

A 123. feladat megoldása:

Kicsinyítsük az AB átmérőjű kört A-ból \frac{AC}{AB} arányban! Ekkor a D pont képe E, AB kör D-beli érintőjének képe pedig AC kör E-beli érintője lesz; e két érintő (a kicsinyítés tulajdonságaiból adódóan) párhuzamos.

A 120-as feladat megoldásából azonban ismert, hogy a rajzolt elrendezésben az AB kör D-beli érintője párhuzamos az AC és a CB körök közös külső érintőjével. Eszerint AC kör imént előállt (E pontbeli) érintője ezzel a közös érintővel egybeesik.

(Természetesen elindulhattunk volna az AB kört C-ből \frac{CB}{AB} arányben kicsinyítve is.)

Előzmény: [854] BohnerGéza, 2007-09-05 23:57:21
[855] SmallPotato2007-09-06 07:09:11

No, majdnem lecsaptam gyorsan a feladatra ... de várok, hátha nálam érdemesebbek írhatják meg. :-)

Előzmény: [854] BohnerGéza, 2007-09-05 23:57:21
[854] BohnerGéza2007-09-05 23:57:21

123. feladat: Bizonyítandó, hogy az AD és BD az érintési pontokat metszi ki a kisebb körökből. ( CD merőleges AB-re. )

Előzmény: [851] SmallPotato, 2007-08-29 23:34:51
[853] sakkmath2007-08-30 09:20:48

Köszönöm, hogy foglalkoztál a feladattal és feltetted a megoldásodat, ami természetesen helyes.

A feladat egyébként - elemi módon - még (legalább) két másik módszerrel is megoldható. Tehát nincs még lelőve semmi, aki esetleg új megoldásra bukkan, nyugodtan tegye fel...

Előzmény: [851] SmallPotato, 2007-08-29 23:34:51
[852] SmallPotato2007-08-29 23:36:56

"808-as" helyett természetesen a 120-as feladat értendő, bocs. :-/

Előzmény: [851] SmallPotato, 2007-08-29 23:34:51
[851] SmallPotato2007-08-29 23:34:51

A 808-asra hoznék egy megoldást; úgy látom, eddig még nem érkezett.

Legyen \overline{AC}=2r, és \overline{CB}=2R.

Húzzunk párhuzamost t2-vel az OAC ponton át; messe ez a BC átmérőjű körnek a t2-vel való érintési pontjába húzott sugarát az E pontban. Ekkor \overline{O_{CB}E} = R-r és az OACOCBE háromszög E szöge derékszög, átfogója pedig R+r.

Az OABDC háromszög C szöge derékszögű, \overline{O_{AB}D} átfogója (lévén az \overline{AB} átmérőjű kör sugara) szintén R+r, és \overline{CO_{AB}} befogója szintén R-r, tehát

az OACOCBE és a OABDC háromszögek egybevágók.

Eszerint viszont COABD\angle=OACOCBE\angle, azaz \overline{O_{AB}D}||\overline{O_{CB}E}, és mivel az érintők merőlegesek a megfelelő sugarakra, t1||t2.

Előzmény: [808] sakkmath, 2007-07-16 18:26:43
[850] zizibi2007-08-24 18:31:31

Köszönöm, segítséget. Így már értem.

Az az igazság ogy a 10. volt már majd 10 éve volt, de nem is azzal van a gond csak nem esett le az egyenlő együtthatók módszer.

Mégegyszer köszönöm! :)

Előzmény: [849] BohnerGéza, 2007-08-24 16:29:24
[849] BohnerGéza2007-08-24 16:29:24

A számolásnál az egyenlő együttgatók módszerét használtam. Ha elvégezted volna a jelzett számolást, észrevetted volna, hogy y kiesik.

Próbáld ki! Az első egyenletet v2-vel, a másodikat v1-gyel szorozva, a kettőben x ua-szor lesz. Kivonva egymásból a kapott egyenleteket x kiesik, y kiszámolható.

Természetesen behelyettesítéssel is kijön ue.

Ezeket legalább tizedikes ismerőseidtől is megtudhattad volna.

Előzmény: [848] zizibi, 2007-08-24 09:41:49
[848] zizibi2007-08-24 09:41:49

Hát igen, én bonyolultabban akartam megoldani :)

De azt nem igazán értem, hogy ha megszorzom az egyik egyenletet v1 a másikat v2-vel, akkor hova tűnik az y? Mert hát ha jól emlékszem, hiányos matektudásomban az van, hogy minden tagot szorozni kell, vagy nem? Ez valamiféle egyszerűsítés?

Elnézést, hogy ennyit értetlenkedem, de szeretném megérteni hátha jobban felfogom...

Előzmény: [846] BohnerGéza, 2007-08-23 14:09:27
[847] Csimby2007-08-24 01:17:04

Volt egy kis elírás a végén: BO=2OF helyesen. (csak hogy nehogy valakit összezavarjon) Szép megoldás!

Előzmény: [844] BohnerGéza, 2007-08-23 03:23:25
[846] BohnerGéza2007-08-23 14:09:27
Előzmény: [845] zizibi, 2007-08-23 11:33:56
[845] zizibi2007-08-23 11:33:56

Igen, erre az egyenletre rá jöttem én is, de kéne még egy egyenlet, gondolom, amiből ki lehet fejezni az x-t és egy másik amiből az y-t.

Na idáig nem jutottam el. Vagy az egyik koordinát egyenletét be lehetne helyettesíteni a másikba, pl. x-et kifejezve belőle?

Előzmény: [842] BohnerGéza, 2007-08-22 20:45:22
[844] BohnerGéza2007-08-23 03:23:25

Ajánlom Gerőcs László cikkét, a KöMaL bemutatkozó oldaláról is elérhető, vagy innen:

http://www.komal.hu/hirek/vegyes/gerocslaszlo-quovadis.h.shtml

Előzmény: [838] Bauer Gábor, 2007-08-21 09:53:03
[843] BohnerGéza2007-08-22 20:48:23

A Pithagórasz-tétel a két pont távolságához (d) kellett.

Előzmény: [841] zizibi, 2007-08-22 14:26:32
[842] BohnerGéza2007-08-22 20:45:22
Előzmény: [841] zizibi, 2007-08-22 14:26:32
[841] zizibi2007-08-22 14:26:32

Kedves BohnerGéza!

Még egyszer köszönöm az előző segítséget, remekül tudom használni (bár koordinátarsz-re nem volt szükségem, mert karakteresen működik a program), de nem igazán értem, hogy Pithagórasz-tétel hogyan is működik ebben az esetben.

Közben pedig újabb problémába ütköztem.

Szintén koordináta jellegü és az előző feladat megfordítása, vagyis adott AB egyenes és egy X pont. Meg szeretném tudni, hogy az X pont milyen merőleges távolságra van A-tól, AB egyenesen mérve illetve milyen messze van AB egyenestől.

Szögfügvényekkel találtam megoldást, de nekem jobban tetszene egy, az előzőhöz hasonló megoldás.

Előre is köszönöm a segítséget!

Előzmény: [836] BohnerGéza, 2007-08-16 21:56:13
[840] jonas2007-08-21 22:27:55

Kell szerkeszteni egy kört, elharmadolni, és érintőket szerkeszteni a megfelelő pontokba.

Előzmény: [838] Bauer Gábor, 2007-08-21 09:53:03
[839] SÁkos2007-08-21 10:53:59

A szabályos háromszögben m=3r és m=\frac{\sqrt3}2a. Így a=2\sqrt3r.\sqrt3ra Pithagorasz-tétel segítségével szerkeszthető.

Előzmény: [838] Bauer Gábor, 2007-08-21 09:53:03
[838] Bauer Gábor2007-08-21 09:53:03

Tisztelt Barátaim! Alapvetó problémával kerültem szembe a következő feadattal: "Szerkesszünk szabályos háromszöget ha adott a beírt kör sugara." Kérdésem a következő: Melyek azok a tételek amelyek átgondolásával a feladat megoldható és mi a szerkesztés menete? Válaszukat hálásan köszönöm.

[837] zizibi2007-08-16 23:00:25

Köszönöm a gyors és kimerítő választ! Még fel kell fognom, de aszem értem!

Hiába, csoda világ a matek! :)

Előzmény: [836] BohnerGéza, 2007-08-16 21:56:13
[836] BohnerGéza2007-08-16 21:56:13
Előzmény: [835] zizibi, 2007-08-16 14:07:12
[835] zizibi2007-08-16 14:07:12

Sziasztok!

Átböngésztem az eddigi hozzászólásokat, de sajnos nem találtam megoldást a problémára, aszem.

Programot szeretnék irni, de a dolgok matematikai oldalával nem igazán vagyok tisztában. Adott, korrdinátásan, két pont által meghatározott vonal. Szeretnék megkeresni egy pontot ami X távolságra van az egyik végétől rajta az egyenesen.

A másik program az lenne, hogy ugyanezen vonal egyik pontjától X távolságra és Y távolságra merőlegesen magától vonaltól lenne a pont.

Előre is köszönöm a segítséget!

[834] BohnerGéza2007-08-09 22:03:01
Előzmény: [833] BohnerGéza, 2007-08-07 20:32:44
[833] BohnerGéza2007-08-07 20:32:44
Előzmény: [832] HoA, 2007-07-31 13:30:57
[832] HoA2007-07-31 13:30:57

Úgy látszik nyári szünet van... Tehát a javaslatot felhasználó megoldás: Mivel A1M=KB1+KC1 , A1M -nek az a oldalra vett A1Ma vetületét állítsuk elő mint KB1 és KC1 A1Ba és A1Ca vetületeinek előjeles összegét. Mivel KB1 merőleges b-re, A1Ba=\varrho.sin\gamma . Hasonlóan, mivel KC1 merőleges c-re, A1Ca=\varrho.sin\beta . Innen A1Ma=\varrho(sin\beta-sin\gamma) . Ugyanígy B1Mb=\varrho(sin\alpha-sin\gamma) . \frac{A_1M_a}{B_1M_b} = \frac{sin \beta  - sin \gamma }{sin \alpha  - sin \gamma } A szinusz tétel többszöri felhasználásával  = \frac{sin \beta - \frac {c}{b}sin \beta  }{sin \alpha  - \frac {c}{a}sin \alpha   } = \frac{sin \beta }{sin \alpha } \frac {1 - \frac {c}{b}}{1 - \frac {c}{a}} = \frac {b}{a} \frac {\frac {b-c}{b}}{\frac{a-c}{c}} = \frac {b-c}{a-c} Ez viszont [806] szerint megegyezik az \frac {O_aA_1}{O_bB_1} aránnyal

Előzmény: [807] BohnerGéza, 2007-07-14 01:45:23
[831] HoA2007-07-30 11:43:17

Ha van az enyémnél kevésbé hosszadalmas, szemléletes megoldásod, örömmel látnám. Javaslatod arra utal, hogy igen. Megpróbáltam a leírt tényt kihasználni, de megint csak az Euler egyenesre jutottam:

M - C = 2 ( ( A + B ) / 2 - O ) ; M - C = A + B - 2O ; M = A + B + C -2O = 3S - 2O

és mivel a jobb oldalon az együtthatók összege 1, ez éppen azt jelenti, hogy M, S és O egy egyenesen van.

Előzmény: [807] BohnerGéza, 2007-07-14 01:45:23
[830] epsilon2007-07-28 13:42:58

Ok Cckek, ebben teljesen igazad van, megpróbálom a feladatot egészében "emésztgeni" Kösz, üdv: epsilon

[829] Cckek2007-07-28 10:41:02

Mivel a függvény folytonos, úgy Darboux tulajdonságú, az-az \forall \lambda\in [\frac{1}{3},1],\exists x_{\lambda}\in [0,1] úgy hogy f(x_{\lambda})=\lambda

Előzmény: [828] epsilon, 2007-07-28 07:08:13
[828] epsilon2007-07-28 07:08:13

Heló Károly és Cckek! Valójában az 1/3 egy elfajult eset, amikor pontok egybeesnek. Ha a 12 pont mind különböző, akkor az az érzésem, hogy 1/3-hoz tetszőlegesen közel lehet (?) de azt nem veszi fel? A folytonossággal értem, hogy hamar lelőhető, de többváltozó esetén van a globális meg komponensenkénti folytonosság, ezzel ezt akarom mondani, hogy még nem elég megggyőző számomra, hogy valóban bármely y az [1/3;1] értékre létezik a 12 pont kérdéses elhelyezkedése. De mindenképpen még gondolkodom, és kösz! Üdv: epsilon

[827] Cckek2007-07-27 22:59:43

A 825-ös hozzászólást nem kell figyelembe venni.:D

Előzmény: [825] Cckek, 2007-07-27 22:46:21
[826] Cckek2007-07-27 22:53:34

Minden értéket felvehet a [\frac{1}{3},1]-ből:) Ugyanis az egy csúcsban összefutó három él által meghatározott tetraéder térfogata,\frac{x_1x_2x_3}{6} , ahol x1,x2,x3 az illető éleken felvett pontok csúcsoktól mért távolsága,a kocka térfogatának (ami 1) és nyolc ilyen tetraéder térfogata összegének a különbsége adja a konvex burok térfogatát. Tehát lesz egy 12 változós függvényünk, mely folytonos, és mint már Károly bebizonyította felveszi a \frac{1}{3} és az 1 értékeket tehát minden értéket [\frac{1}{3},1]-ből.

Előzmény: [825] Cckek, 2007-07-27 22:46:21
[825] Cckek2007-07-27 22:46:21

Minden értéket felvehet:) Ugyanis az egy csúcsban összefutó három él által meghatározott tetraéder térfogata, \frac{x_1x_2x_3}{6}, ahol x1,x2,x3 az illető éleken felvett pontok csúcsoktól mért távolsága,a kocka térfogatának (ami 1) és nyolc ilyen tetraéder térfogata összegének a különbsége adja a konvex burok térfogatát. Tehát lesz egy 12 változós függvényünk, mely folytonos, és mint már Károly bebizonyította felveszi a 0 és az 1 értékeket tehát minden értéket [0,1]-ből.

Előzmény: [824] epsilon, 2007-07-27 17:54:59
[824] epsilon2007-07-27 17:54:59

Köszi Károly! Ilyesmi érdekelne, hogy a konvex burok térfogata lehet pl. 2/3, továbbá milyen értékeket vehet fel a (0,1) intervallumban. Előreis kösz, üdv: epsilon

[823] Hajba Károly2007-07-26 22:58:55

Ha minden pont oly módon van valamely csúcsban, hogy minden csúcsra kerül pont, akkor a konvex burok nyilvánvalóan megegyezik a kockával. Ha a kocka térfogatát egységnyinek veszem, akkor a burok maximális térfogata 1 egység.

Ha a pontokat úgy próbáljuk a csúcsokan elhelyezni, hogy minél kevesebb csúcson jelenjenek meg, akkor a 4 csúcs a lapok átellenes (átlós) csúcsaira kerülnek, s a 4 csúcs egy szabályos tetraédert alkot. Ekkor a burok térfogata 1/3 egység.

Nem erősségem a bizonyítás, de gyakorlatilag ez utóbbi a minimális térfogat.

Előzmény: [821] epsilon, 2007-07-26 14:06:03
[822] nadorp2007-07-26 14:28:01

Igazad van, viszont kör esetén ( bár ez a legspeciálisabb eset) szerintem lehetne valamit keresni.

Előzmény: [815] BohnerGéza, 2007-07-22 19:59:21
[821] epsilon2007-07-26 14:06:03

Helló Károly! Igen, úgy van ahogyan mondod. Kiváltképpen az érdekelne, hogy ennek a konvex buroknak a térfogatának az értéke a kocka térfogatának hányad részével lehet egyenlő, és esetleg mely értékeket nem veheti fel a kocka térfogatából? Üdv: epsilon

Előzmény: [820] Hajba Károly, 2007-07-24 23:50:31
[820] Hajba Károly2007-07-24 23:50:31

Üdv!

Egy naív kérdés, ellenőrzésképpen, hogy jól értem-e.

A 12 pont konvex burka az a kockából visszamaradó idom, melyet úgy kapunk, hogy a 8 sarkából tetraédereket levágunk, s egy-egy szomszédos csúcshoz tartozó tetraéderek megfelelő csúcspontjaik a kijelölt pontban azonosak?

Ha igen, akkor a következő -vizuális úton nyert- meglátásaim vannak: Ekkor az idom mindenképpen konvex. Szélsőséges esetben legfeljebb három pont a csúcsban lehet azonos is. Maximális térfogata egységnyi, míg minimális térfogata harmad egységnyi.

Előzmény: [816] epsilon, 2007-07-24 09:16:24
[819] epsilon2007-07-24 21:55:31

Helló Cckek! Itt vannak nekem is kételyeim, hogy minden esetben felvehető-e úgy a 12 pont, hogy a szóbanforgó test konvex legyen? Na persze, ha még térfogati megszorítások, kitétek is bekerülnek, akkor még áltáthatatlanabb. Nyilván a "konvex burok" csak ekkor illik a szóhasználatra!

Előzmény: [817] Cckek, 2007-07-24 10:09:12
[818] jonas2007-07-24 11:24:51

Ez egy másik feladatra emlékeztet a Pólya-Szegő analízis feladatgyűjteményből, amit feladtunk a szemináriumon. Sajnos miután utánanéztem, kiderült, hogy nem segít ehhez a feladathoz, mert a térfogatról nem mond semmit. Ide másolom a feladatsorról

Legyen a 3-térben egy test olyan tulajdonságú, hogy szakaszban vagy pontban (vagy az üres halmazban) metsz minden, a koordinátatengelyek bármelyikével párhuzamos egyenest. Legyen a test vetülete a koordinátasíkokra P, Q, illetve R területű. Legyen a test felszíne A. Lássuk be, hogy


2 \sqrt{P^2 + Q^2 + R^2} \le A \le 2 (P + Q + R)

Igazoljuk, hogy a becslés éles.

Előzmény: [816] epsilon, 2007-07-24 09:16:24
[817] Cckek2007-07-24 10:09:12

A pontok által meghatározott test, ha a pontokat megfelelő sorrendben kötjük össze konvex lesz nem?

Előzmény: [816] epsilon, 2007-07-24 09:16:24
[816] epsilon2007-07-24 09:16:24

Helló! A következő kérdésem lenne: ha egy kocka minden élén felveszünk 1-1 pontot, mi lesz a 12 pont konvex burka, esetleg meg lehetne-e mondani a térfogatának szélsőértékeit, vagy azokat az értékeket amiket ez felvehet ? Előre is kösz Mindenkinek! Üdv: epsilon

[815] BohnerGéza2007-07-22 19:59:21

Az előző hozászólásomban helyesen a 122. feladat kell legyen.

Elnézést kérek ,,Lajos Arpad´´-tól, akinek érdekes a 121. feladata. Annak megoldására kiváncsi vagyok. Ebben a hőségben inkább azt hiszem, nem speciális esetben nincs rá olyan képlet, melybe egy pont koordinátáját helyettesítve megkapjuk, hogy az fényes vagy sem.

Előzmény: [814] BohnerGéza, 2007-07-22 19:22:10
[814] BohnerGéza2007-07-22 19:22:10

121. feladat: A 120. feladat alapján szerkesztendő két egymást kívülről érintő kör közös külső érintője. (Indoklása egyben a 120. feladat megoldása is lehet.)

Előzmény: [808] sakkmath, 2007-07-16 18:26:43
[813] Lajos Arpad2007-07-21 10:23:16

A 121. feladat Hmmm... Igazad van. Ezek szerint nem volt olyan nagy kudarc hogy nekem nem sikerült ;). Nekem csak egy szép integrál jött ki, amit nem igazán tudtam megoldani annak ellenére, hogy próbálkoztam egy ideig. Egy érdekes feladat: Adott egy origó középpontú ellipszis, egyenlete: (x/a) 2 + (y/b) 2 = 1 Ezen az ellipszisen teljes visszaverôdés figyelhetô meg. Az ellipszis egyik pontjáról valamilyen irányba(az ellipszis belseje felé) elindítunk egy fénysugarat, melynek szélessége elhanyagolható, és a végtelenségig verôdik az ellipszis falán vissza. A kiindulópont legyen M(x0,y0), az irány pedig alfa az origóhoz képest. A fénysugár a végtelenségig verôdik vissza az ellipszis falán. A kérdés az, hogy (a,b,X0,Y0,alfa) függvényében hol lesz fényes az ellipszis belseje? Bevallom ôszintén, hogy én ezen a problémán kb. 3 hónapig gondolkodtam hetedmagammal ameddig egy valamennyire elfogadható megoldást találtam. Jelölés :  : hatvány

[812] ágica2007-07-21 09:39:22

Hm.. ezt miből gondolod? Tudtommal az ellipszis kerületét nem lehet ilyen szépen kifejezni. Lásd: http://mathworld.wolfram.com/Ellipse.html

Előzmény: [811] Lajos Arpad, 2007-07-21 06:54:48
[811] Lajos Arpad2007-07-21 06:54:48

elnézést, a kerület pi(a+b)

[810] Lajos Arpad2007-07-21 06:54:06

üdv mindenkinek, lenne egy kérdésem. Tudjuk hogy az ellipszis kerülete=pi*a*b, ahol a az ellipszis szélessége "a", hosszúsága pedig "b".Hogyan kéne bizonyítani? Meg próbáltam oldani a hozzá tartozó elliptikus integrált, de beletörött a bicskám.

[809] lorantfy2007-07-17 14:38:56

Szép megoldás. Grat!

Előzmény: [806] HoA, 2007-07-13 17:12:29
[808] sakkmath2007-07-16 18:26:43

A 120. feladat (ujjgyakorlat is lehetne):

Legyen adott az AB szakasz, melynek tetszőleges belső pontja C. Az AB, AC, CB szakaszok, mint átmérők fölé - azonos partjukra - félköröket rajzoltunk. Az AB-re C-ben állított merőleges D-ben metszi az AB félkört. Az AB félkör D-hez tartozó érintője t1, a másik két félkör közös (CD-től különböző) érintője t2. Bizonyítsuk be, hogy t1 párhuzamos t2-vel.

[807] BohnerGéza2007-07-14 01:45:23

Köszönöm HoA szép megoldását! Egy kiegészítés és egy javaslat még:

A 119. feladat feltételei, "ha ezek léteznek és nem nullák", miatt [806]-ban a megoldást így kezdtem volna: Egyenlő szárú háromszögben az állítás igaz, különben ... . Természetesen ez nyilvánvaló, de versenyen akár helyezés múlhat azon, ha leírjuk.

Javaslom, hogy a 118. feladatban szereplő eredeti 3 pontnak az ABC valamelyik oldalára eső vetületére számoljuk ki az osztóviszonyt róval és r-rel. ( beírt, körülírt kör sugara ) Használjuk ki, hogy egy csúcsból a magasságpontba mutató vektor kétszerese a körülírt kör kp-jából a csúccsal szemközti oldalfelező pontba mutató vektornak!

Előzmény: [806] HoA, 2007-07-13 17:12:29
[806] HoA2007-07-13 17:12:29

118. feladat: [803] és [805] alapján elég azt megmutatni, hogy az O, K pontoknak és az A1B1C1\Delta S súlypontjának az a illetve b oldalakra eső Oa,A1,Sa és Ob,B1Sb vetületeire teljesül: \frac {O_aS_a}{O_bS_b} = \frac {O_aA_1}{O_bB_1} . COa=a/2,CA1=s-c, így OaA1=s-c-a/2=(b-c)/2. hasonlóan ObB1=(a-c)/2 . A kettő hányadosa (b-c)/(a-c)

CSb felírható mint a CA1,CB1ésCC1 szakaszok b-re eső vetületeinek számtani közepe, így ObSb=CSb-COb=1/3((s-c).cos\gamma+(s-c)+a.cos\gamma+(b-a.cos\gamma)*((s-a)/c))-b/2. Itt 2s = a+b+c helyettesítéssel cos\gamma-t a koszinusztételből kifejezve átrendezéssel adódik:

 O_bS_b =\frac {a-c}{12abc}\bigg\{ a^2b + ab^2 - a^3 - b^3 + c ( b^2 + 4ab + a^2 ) + c^2 ( a + b ) - c^3 \bigg\}

OaSa ebből a és b szerepének felcserélésével kapható. Látható , hogy a nagyzárójelben a és b szimmetrikusan lép fel, a kifejezésnek ez a része változatlan, tehát

 O_aS_a =\frac {b-c}{12abc}\bigg\{ a^2b + ab^2 - a^3 - b^3 + c ( b^2 + 4ab + a^2 ) + c^2 ( a + b ) - c^3 \bigg\}

A kettő hányadosa \frac{O_aS_a}{O_bS_b} = (b-c)/(a-c) = \frac {O_aA_1}{O_bB_1}

Előzmény: [801] BohnerGéza, 2007-06-30 23:50:41
[805] HoA2007-07-09 13:52:12

Még egy segítség: Mivel K az A1,B1,C1\Delta körülírt körének középpontja, KM A1,B1,C1\Delta Euler-egyenese, amin A1,B1,C1\DeltaS1 súlypontja is rajta van. Tehát elég belátnunk, hogy O,KésS1 egy egyenesen vannak.

Előzmény: [801] BohnerGéza, 2007-06-30 23:50:41
[804] lorantfy2007-07-03 14:56:00

Ha Q pont nincs rajta a PR egyenesen, akkor QaQ egyenesnek PR egyenessel való metszéspontja legye A, QbQ és PR metszéspontja pedig B. Ekkor \frac{PR_a}{Q_aR_a}=\frac{PA}{AR}\ne \frac{PB}{BR}=\frac{PR_b}{Q_bR_b} vagyis (PaQaRa)\ne(PbQbRb).

Előzmény: [803] lorantfy, 2007-07-03 14:35:02
[803] lorantfy2007-07-03 14:35:02

119. feladathoz: Toljuk el a és b egyeneseket önmagukkal párhuzamosan úgy, hogy P ponton menjenek át. Ekkor, ha Q a PR egyenesen van, akkor a párhuzamos szelők tételéből következik, hogy  \frac{PR_a}{Q_aR_a}=\frac{PR}{QR}=\frac{PR_b}{Q_bR_b} és mivel Pa=P=Pb ezért (Pa Qa Ra) = (Pb Qb Rb).

Előzmény: [801] BohnerGéza, 2007-06-30 23:50:41
[802] lorantfy2007-07-01 14:30:12
Előzmény: [801] BohnerGéza, 2007-06-30 23:50:41
[801] BohnerGéza2007-06-30 23:50:41

118. feladat: A Gillis - Turán Matematikaverseny egyik szép feladatát ajánlom:

Legyen ABC egy olyan háromszög, ami nem egyenlő oldalú. Az ABC háromszög beírt köre érintse a megfelelő oldalakat az A1, B1, C1 pontokban, és legyen M az A1B1C1 háromszög magasságpontja. Bizonyítsuk be, hogy M rajta van az ABC háromszög körülírt körének középpontját és beírt körének középpontját összekötő egyenesen.

A megoldáshoz javaslom a következő segítséget:

119. feladat: Legyen a P, Q és R pont merőleges vetülete az a ill. b nem párhuzamos egyenesen Pa, Qa és Ra ill. Pb, Qb és Rb. Bizonyítandó, hogy (Pa Qa Ra) = (Pb Qb Rb) -ha ezek léteznek és nem nullák- csakkor igaz, ha a P, Q és R egy egyenesen van.

/ Az egy egyenesen lévő A, B és C pontok esetén ezek osztóviszonyán értjük az (A B C)=AC/BC előjeles hányadost. /

[800] lorantfy2007-06-29 15:03:41

Kedves Levente!

Ügyes felvetés volt, Matyinak dicséret érte. Látom C gyakorlatokat küldött az idén. Bátorítsd egy kicsit, hogy rendszeresebben küldjön!

Üdv! Laci

Előzmény: [799] klevente, 2007-06-29 11:49:08
[799] klevente2007-06-29 11:49:08

Kedves hozzászólók és megoldók!

A "tényleg ellipszis-e a mértani hely?" témát én vetettem volt fel. Köszönöm az érdeklődést és a problémát kimerítő megoldást.

A történet kezdetéhez annyit tennék hozzá, hogy az Árpád Gimnáziumban épp a Simson-egyenest tanítottam 9. osztályban specmaton, és a Kiss Matyi nevű (egyébként is elég tehetséges) diák állt elő ezzel a sejtéssel a GeoGebrát nézegetve. Elég ügyes intuíció, azt hiszem.

Koncz Levente

Előzmény: [776] lorantfy, 2007-06-18 14:46:29
[798] jonas2007-06-27 21:04:13

Nos, ezeket a bolhás feladatokat az egyetemen hallottam Feladatmegoldó szemináriumon, azaz legalább négy éve. Egy másik sorozat bolhás feladat (amit most nem lövök le itt) állítólag Pósa Lajostól származik, ezért el tudom képzelni, hogy ezek is.

Előzmény: [793] Hajba Károly, 2007-06-27 00:05:16
[797] Python2007-06-27 15:36:56

Utolosó matekórák egyikén n=4-re láttam a feladatot, és én térbe általánosítottam:

Van 8 , az (elég gyenge) ábrán látható derékszögű triédert bevilágító, rögzített csúcsú pontszerű, átlátszó lámpánk a térben , be lehet e velük a teret világítani forgatással?

Az általános megoldásról annyit, hogy ha felvehetünk egy pontot, és abból sugarasan 360o/n fokonként félegyenest indítva, bármely 2 szomszédos félegyenes közti szögben van fényforrás, akkor n-re igaz.

(Toljunk minden cikket egy nem benne lévő pontba, így az eltolt tartalmazni fogja az eredetit, és így az n eltolt tartalmazza az n eredetit=a teljes síkot.)

Előzmény: [796] Sirpi, 2007-06-27 14:14:27
[796] Sirpi2007-06-27 14:14:27

A fényforrást eltakaró gömbről beugrott egy másik példa, régen n=6-ra volt a kömalban, és nem tudom az általános megoldását:

Igaz-e, hogy ha adott a síkon n db. pont, és mindegyik egy 360o/n nyílásszögű fényforrás kezdőpontja, akkor a fényforrások beforgathatók úgy, hogy a teljes síkot bevilágítsák? A fényforrások pontszerűek és átlátszóak, azaz ha rávilágítunk egy fényforrásra, akkor az nem fogja a maga mögötti félegyenest kitakarni.

Kicsi és kerek számokra kiküzdöttem (8-ig nincs meg 5-re és 7-re, amire megvan, azokra nyilván igaz), de nagyon örülnék egy általános megoldásnak.

[795] Hajba Károly2007-06-27 08:18:51

Beszéljünk inkább műltidőről, legalább is ami a nagy gondolkodásokról szól. :o)

Először az Index idézett fórumát látogattam, ami időnként fölpörög ill. gyengén elindult egy hasonló az Origón, de az nem tudott fölpörögni azóta sem. Majd jött a KöMaL, amire az Origón korábban sűrűn megjelent Géza (Qeza) hívta fel a figyelmem.

De biztos van még más matekos fórum is amiről nem tudok.

Előzmény: [794] Csimby, 2007-06-27 01:29:42
[794] Csimby2007-06-27 01:29:42

Te hány matekos fórumba írogatsz? :-)

Előzmény: [793] Hajba Károly, 2007-06-27 00:05:16
[793] Hajba Károly2007-06-27 00:05:16

Ez a vonalmenti bolhás feladat vagy 2 éve szerepelt már az Indexen:

Bolhás feladat

Előzmény: [790] jonas, 2007-06-25 20:12:37
[792] Sirpi2007-06-26 11:39:49

Kösz a gratulációkat, nem gondoltam volna, hogy ekkora ötlettel állok elő :-)

A bolhás feladatnál pedig ekvivalens feladato(ka)t kapunk, ha a minden lépés utáni egységszakasz/négyzet lecsapása helyett az első lépés után csaphatunk végtelen sokszor, viszont rendre 1, 1/2, 1/3 stb. (oldal)hosszúságú szakaszokat/négyzeteket.

Mivel \sum 1/n divergens, \sum 1/n^2 pedig konvergens, ezért 1 dimenzióban elkapható a bolha (a módosított feladatban csapjunk mondjuk a [0;1],[-1/2;0],[1;4/3],[-3/4,-1/2] stb. szakaszokra), kettő vagy magasabb dimenzióban pedig nem.

Előzmény: [790] jonas, 2007-06-25 20:12:37
[791] Hajba Károly2007-06-25 23:29:30

Először meg is hökkentem az alacsony számon, épp rá is akartam kérdezni a megoldásra, de még időben bevillant nekem is a tetraéderes megoldás, hogy ne küldjem el. :o)

Te rögtön rájöttél, nekem kellett a szám ismerete is. Én is gratulálok.

Előzmény: [788] Sirpi, 2007-06-24 09:23:44
[790] jonas2007-06-25 20:12:37

Na, ez a feladat a bolhás feladatokra emlékeztet.

(A számegyenesen ugrál egy bolha az origóból indulva. Minden lépésben ugyanakkorát ugrik és ugyanarra, de hogy mekkorát, az egy tetszőleges ismeretlen valós szám. Minden lépés után (miután halljuk a koppanást) rácsaphatunk a számegyenes egy tetszőleges, egység hosszú intervallumra. (A bolha olyan kicsi, hogy nem lehet látni.) Van-e olyan stratégia, amivel biztosan le lehet csapni a bolhát előbb-utóbb?

A második változatban a bolha a síkon ugrál az origóból indulva, minden lépésben ugyanazzal az ismeretlen valós vektorral mozdul el az előző lépéshez képest. Minden lépés után egy egységnégyzetre lehet rácsapni. El lehet-e kapni a bolhát.)

Előzmény: [785] Cckek, 2007-06-20 16:13:21
[789] Cckek2007-06-24 12:11:17

Mindenesetre a tetraéderes ötlet zseniális, gratulálok:)

Előzmény: [788] Sirpi, 2007-06-24 09:23:44
[788] Sirpi2007-06-24 09:23:44

Hm, ez izgalmasabban hangzik, de erre a részére egyelőre nem tudom a megoldást. Viszont mivel az eredeti kérdésre jó pár napja beírtam az eredményt, és így mindenkinek volt ideje picit gondolodni rajta, aki akart, ezért írok hozzá egy megoldásvázlatot is (tényleg csak vázlat, nem bizonyítok benne minden apró részletet):

Két dolgot kell bizonyítani:

a) 3 gömb nem elég

Illesszünk síkot a 3 gömbközéppontra, majd a fényforrásban állítsunk merőlegest erre a síkra. Ennek az egyenesnek az a fényforrásból kiinduló nyílt félegyenese, amelyik nem metszi a síkot, nyilvánvalóan nincs eltakarva (mert ha bármelyik pontja része lenne valamelyik gömbnek, akkor maga a fényforrás is).

b) 4 már elég

Helyezzük bele a fényforrást egy tetraéder belsejébe. Ha most minden lapot eltakarunk a fényforrás szemszögéből, akkor készen is vagyunk. Arra kell csak vigyázni, hogy ezek a gömbök ne metsszék egymást, de ez a gömbök megfelelő eltávolításával és ezzel arányos megnagyobbításával megtehető (egymás után tesszük le a gömböket és mindegyiket úgy távolítjuk-nagyítjuk középpontos hasonlósággal a fényforrásból, hogy távolabbra kerüljön, mint a lefedendő térszögbe belógó más gömbrészek).

Előzmény: [785] Cckek, 2007-06-20 16:13:21
[787] lorantfy2007-06-22 15:26:24

Szia HoA!

Persze, jól gondolod.

A Word-höz van MathType egyenletszerkesztőm, azzal jóval gyorsabban lehet egyenleteket írni, mint TeX-ben.

Mikor készen van kifényképezem a vágólapra Print Screen-nel aztán Paint-ben gif formátumban elmentem.

Még átlátszóvá lehetne tenni, akkor feltöltve a sárga háttérre és nem lenne keretes.

Előzmény: [786] HoA, 2007-06-22 09:58:39
[786] HoA2007-06-22 09:58:39

Milyen programmal állítod elő ezt az ízléses dokumentum formátumot? Jól gondolom, hogy ez a doku képernyőfotója?

Előzmény: [774] lorantfy, 2007-06-14 15:55:42
[785] Cckek2007-06-20 16:13:21

Helyes. És ha a gömbök sugara egyenlő?

Előzmény: [784] Sirpi, 2007-06-19 23:23:14
[784] Sirpi2007-06-19 23:23:14

4

Előzmény: [782] Cckek, 2007-06-18 21:42:27
[783] BohnerGéza2007-06-19 20:13:17

Valóban, HoA-nak igaza van, figyelmetlen voltam, a befogó-tétel adja f-et!

Lehetne szögfv-ek nélkül, de röviden:

Előzmény: [777] HoA, 2007-06-18 16:25:53
[782] Cckek2007-06-18 21:42:27

Modosítok. A gömbök diszjunktak természetesen.

[781] jonas2007-06-18 21:21:04

Különben mindegy, két gömb elég, ha érintik egymást, és az érintési pontban van a fényforrás.

Előzmény: [779] Cckek, 2007-06-18 18:28:14
[780] jonas2007-06-18 21:20:05

A gömbök metszhetik egymást?

Előzmény: [779] Cckek, 2007-06-18 18:28:14
[779] Cckek2007-06-18 18:28:14

Legalább hány gömb képes teljesen leárnyékolni egy pontszerű fényforrást? (a fényforrás nem lehet a gömbök belsejében)

[778] HoA2007-06-18 16:31:00

Az előző hozzászólás utolsó mondatai természetesen úgy értendők, hogy itt Q nem az ellipszis pont, hanem az F - ből e-re bocsátott merőleges és a Thálesz-kör metszéspontja.

Előzmény: [777] HoA, 2007-06-18 16:25:53
[777] HoA2007-06-18 16:25:53

116.és117.feladat: Két "segédtétel" : 1) Az x tengely B pontjából merőlegest bocsátunk az O-n átmenő m meredekségű egyenesre, a talppont legyen C. C merőleges vetülete az x tengelyre A. Ekkor \frac{OA}{OB} = \frac{1}{1+m^2} . Jelöljük ezt az arányt q-val. 2) Tekintsük ismertnek az alábbi ellipszis pontokat eredményező eljárást: A közös O középpontú k1 és k2 köröket metsző O-n áthaladó e egyenest O körül forgatjuk, a körökkel alkotott metszéspontok P1 ill. P2. A P1-n át az y tengellyel és P2-n át az x tengellyel párhuzamos egyenesek metszéspontja Q. Belátható, hogy az így adódó Q pontok egy ellipszisen találhatók.

Eredeti feladatunk r sugarú k körének P pontjából az O-n átmenő 0<m<1 meredekségű e egyenesre bocsátott me egyenes talpponja legyen M, az x tengellyel alkotott metszéspontja A. A P-ből a -m meredekségű f egyenesre bocsátott mf egyenes talpponja legyen N, az x tengellyel alkotott metszéspontja C. P merőleges vetülete az x tengelyen B. APC \Delta egyenlőszárú, magassága PB, így OB = (OA + OC)/2. Ha M x tengelyre merőleges vetülete D, N-é F és az MN szakasz Q felezőpontjáé E, az 1. segédtétel szerint OF=q.OC, OD=q.OA, így OE=(OD+OF)/2=q.(OA+OC)/2=q.OB. QE és OP metszéspontja legyen R. OR=q.OP, Q rajta van az OP egyenes és az O középpontú, qr sugarú kör R metszéspontjából az x tengelyre bocsátott merőlegesen.

Q -nak az y tengelyre vett vetületét vizsgálva hasonló eredményre jutunk, csak ott az 1. segédtétel szerint fellépő meredekség m helyett 1/m, így az adódó kicsinyítési arány \frac{1}{1+ \frac{1}{m^2}} = \frac{m^2}{1+m^2} = s . Q rajta van az OP egyenes és az O középpontú, sr sugarú kör S metszéspontjából az y tengelyre bocsátott merőlegesen. A Q pontok tehát a 2. segédtételben leírt ellipszis pontjai, ahol a generáló körök sugarai ( az ellipszis tengelyei ) qr illetve sr.

[775] ábráján a T pont az OEP derékszögű \Delta-ben éppen az 1. segédtételben leírt módon jön létre, tehát OT = qr = a. Mivel q+s =1, TE = OE - OT = r ( 1 - q ) = sr = b.

Az f-re vonatkozó állítás téves: Legyen a [775] ábráján OE és FQ metszéspontja U. Nem QU, hanem QO lesz f-el egyenlő. Bizonyítás: Az egységsugarú OE körben OU = OF \cdot cos(2\alpha) = cos^2(\alpha) - sin^2(\alpha) = \frac{1}{1+m^2} - \frac{m^2}{1+m^2} = \frac {1-m^2}{1+m^2} , ahol \alpha az m meredekségű egyenes irányszöge. Az OQE derékszögű \Delta-ben OQ OU és OE = 1 mértani közepe, vagyis OQ^2 = \frac {1-m^2}{1+m^2} . Másrészt az ellipszisben f2=a2-b2, esetünkben q^2 - s^2 = \frac{1}{1+m^2} - \frac{m^2}{1+m^2} = \frac {1-m^2}{1+m^2} . Tehát f2=OQ2 ; f = OQ

Előzmény: [775] BohnerGéza, 2007-06-17 22:24:45
[776] lorantfy2007-06-18 14:46:29

Az ellipszis nagytengelyét r sugarú körnek az x tengellyel való R1 pontból az e és f egyenesekre bocsájtott merőlegesek talppontjait összekötő szakasz K1 felezőpontjának O-től való távolsága adja. Tehát a nagytengely OK1.

A kistengelyt hasonlóan az R2 pontból származtatva OK2 adja.

A Thálesz-körben lévő OEP háromszög egybevágó az OR1N1 és az OR2N2 háromszögekkel, mert szögeik megegyeznek és átfogójuk r hosszú. Tehát megfelelő részháromszögeik is egybevágóak.

Így az ellipszis nagytengelye: a=OK1=OT, kistengelye: b=OK2=TE. Tehát r=a+b.

Előzmény: [775] BohnerGéza, 2007-06-17 22:24:45
[775] BohnerGéza2007-06-17 22:24:45

117. feladat: Használjuk föl, hogy a 116. feladat mértani helye ellipszis! Bizonyítsuk, hogy ennek két (fél)tengelye és a (fél)fókusztávolság megszerkeszthető az ábrán látott módon! ( A [774]-es hozzászólás számolási eredményeinek használata nem szükséges! )

A két adott egyenes e és f, az adott kör sugara egyenlő az ábra OE szakaszával. Válasszuk úgy a jelöléseket, hogy az FOE szög ne legyen nagyobb derékszögnél, messe az OE Thálesz-köre FOE szögfelezőjét P-ben. A P-ből e-re állított merőleges talppontja T. Tehát igazolandó, hogy a=OT, b=TE, valamint az F-ből e-re állított merőleges szakasznak a Thálesz-körbe eső része f!

[774] lorantfy2007-06-14 15:55:42
Előzmény: [773] lorantfy, 2007-06-14 10:59:58
[773] lorantfy2007-06-14 10:59:58
Előzmény: [772] lorantfy, 2007-06-13 16:31:31
[772] lorantfy2007-06-13 16:31:31

Látható, hogy egyik egyenest \vec{MM_1} vektorral eltolva a mértani hely pontjai \vec{FF_1}=\frac{\vec{MM_1}}{2} vektorral tolódnak el. Így aztán legegyszerűbb, ha mindkét egyenes a kör középpontján megy át.

Előzmény: [770] HoA, 2007-06-11 22:13:23
[771] BohnerGéza2007-06-11 22:35:30

116. feladat. Egy ábra az általános esethez:

Előzmény: [769] lorantfy, 2007-06-11 21:08:34
[770] HoA2007-06-11 22:13:23

Mégegy segítség: Hogyan változik a Q pontok mértani helye, ha az egyik egyenest önmagával párhuzamosan d távolsággal eltoljuk?

Előzmény: [767] BohnerGéza, 2007-06-06 22:32:11
[769] lorantfy2007-06-11 21:08:34

Nézzünk először egy spec. esetet. Amikor a két egyenes, AB és BC merőlegesek egymásra. Ekkor a talppontok felezőpontja egyben PB szakasz felezőpontja is. Igy a keresett mértani hely a kör B középpontú felére kicsinyített képe.

Előzmény: [767] BohnerGéza, 2007-06-06 22:32:11
[768] BohnerGéza2007-06-06 22:40:04

Köszönöm HoA szép megoldását!

A [756]. hozzászólásban a teljes indukcióra utaltam. Talán érdemes lehet azzal is végiggondolni a feladatot.

Előzmény: [764] HoA, 2007-05-31 22:11:09
[767] BohnerGéza2007-06-06 22:32:11

Nagyon szép észrevétel, de mielőtt valaki összekapcsolná a Simson-egyenessel:

klevente feladata, 116. feladat: Adott egy kör és két egyenes. A kör P pontjából állítsunk merőlegest az egyenesekre, a talppontok közti felezőpont legyen Q. Bizonyitandó, a Q pontok mértani helye ellipszis, ha P végigfut a körön.

Előzmény: [765] klevente, 2007-06-06 14:17:45
[766] lorantfy2007-06-06 14:54:20

Jó!!!!

Előzmény: [765] klevente, 2007-06-06 14:17:45
[765] klevente2007-06-06 14:17:45

Egy háromszög köré írt kör tetszőleges P pontjából állítsunk merőlegeseket a háromszög oldalegyeneseire. Közismert, hogy ezen merőlegesek talppontjai kollineárisak, ez az ún. Simson-egyenes.

Vizsgáljuk valamelyik két talppont összekötő szakasza felezőpontjának mértani helyét, ha P befutja a köréírt kört. A GeoGebra programmal nézve ez szinte bizonyosan ellipszis.

Kérdések: a) tud-e valaki arról, hogy ez a tény (sejtés) ismertnek számít-e a matematikában? b) tudná-e valaki bizonyítani vagy cáfolni?

Koncz Levente

[764] HoA2007-05-31 22:11:09

114** feladat Vizsgáljuk az li hosszúságú ri rúdra ható erőket. A rúdra hat a feladatban adott Fi külső erő és a csuklókban a szomszédos rudak Gi1 és Gi2 kényszerereje. Bontsuk fel az erőket rúdirányú és a rúdra merőleges összetevőkre. Gi1 és Gi2 rúdirányú összetevői egymás ellentettjei, mert a rúd nyugalomban van és más rúdirányú erő nem hat rá. Gi1 és Gi2 rúdra merőleges összetevői egyenlőek és \frac {F_i}2 nagyságúak, mert a rúd másik végére vonatkozó nyomatékuk li karon egyensúlyt tart Fi erő \frac {l_i} 2 karon kifejtett nyomatékával. Megállapíthatjuk tehát, hogy Gi1 és Gi2 egyenlő nagyságűak. Mivel a csuklókban a rudak azonos nagyságú erővel hatnak egymásra, a szomszédos rúdra hatő kényszererő is akkora, mint Gi1 és Gi2 és így tovább : az összes csuklóban ugyanakkora erők hatnak. Legyen ennek az erőnek a nagysága G.

Tekintsük a szomszédos, li és lj hosszúságú ri és rj rudakra ható erőket. Válasszuk úgy az ábrán az erő mértékegységét, hogy a rúd hosszával arányos Fi és Fj erőket éppen li és lj hosszú vektorral ábrázoljuk. Az ri rúdra a Q csuklóban ható Gi erő rúdra merőleges összetevője Fi/2=li/2 . Rajzoljuk meg Q-ban a sokszög belseje felé mutató, Gi-re merőleges, G hosszúságú Gn vektort, Gi 90 fokos elforgatottját. Ennek ri irányába eső összetevője li/2, tehát O végpontja rajta van ri felező merőlegesén. De a Gn vektort úgy is tekinthetjük, mint az rj rúdra Q-ban ható Gj erö -90 fokos elforgatottját. Mivel Gj erö rj -re merőleges összetevője lj/2 nagyságú, Gn -nek rj irányába eső összetevője is ekkora, vagyis O rajta van rj felező merőlegesén is. Vagyis a P,Q,R pontok egy G sugarú körön vannak. A gondolatmenetet a következő R csuklóra alkalmazva azt kapjuk, hogy a Q,R, S pontok is egy G sugarú körön vannak. A két kör azonos: középpontjuk QR felező merőlegesén a sokszög belseje felé Q-tól és R-től G távolságra lévő O pont. Az eljárást folytatva adódik, hogy a csuklók húrsokszöget alkotnak.

Előzmény: [747] BohnerGéza, 2007-05-23 00:28:23
[763] Lóczi Lajos2007-05-30 01:04:13

Az állítás igazsága egy szép integrálgeometriai képletből rögtön következik. Egy konvex síkbeli alakzat kerületét ugyanis az


\frac{1}{2}\int_0^{2\pi} w(\varphi)d\varphi

integrál adja meg, ahol w(\varphi) az alakzat szélességfüggvénye, azaz w(\varphi) a konvex alakzatot közreszendvicselő és az x-tengellyel \varphi szöget bezáró két párhuzamos egyenes távolsága.

Előzmény: [759] V Laci, 2007-05-29 17:48:47
[761] jonas2007-05-29 23:00:19

Szép bizonyítás.

Előzmény: [762] Sirpi, 2007-05-29 20:43:23
[762] Sirpi2007-05-29 20:43:23

Az állítás igaz. A bizonyítás úgy megy, hogy közelítsük mindkét konvex alakzatot zárt poligonokkal úgy, hogy a belső poligon se metssze a külsőt.

Ezután vegyük a belső sokszög egy élét, és hosszabbítsuk meg mindkét irányban. Az egyenes külső sokszöggel vett két metszéspontja két töröttvonalra osztja a külső sokszöget. Ebből dobjuk el azt, ami nem tartalmazza a belső sokszöget és helyettesítsük a meghosszabbított szakasszal. Ezzel a lépéssel a külső sokszög kerülete csökkent (valamint éleinek száma sem nőtt). Véges sok lépés során eljutunk oda, hogy a levagdosások hatására a külső és a belső sokszög megegyezik.

Azt kaptuk tehát, hogy a belső sokszög minden poligonközelítése kisebb kerületű a külső sokszög bármely poligonközelítésénél, vagyis a belső alakzat kerülete is kisebb a külsőénél.

Előzmény: [759] V Laci, 2007-05-29 17:48:47
[760] s.addam2007-05-29 19:29:33

?

[759] V Laci2007-05-29 17:48:47

Sziasztok!

Ha egy konvex síkbeli alakzat a belsejében tartalmaz egy másik konvex síkbeli alakzatot, akkor a belsőnek a kerülete kisebb mint a külsőnek.

Igaz-e az állítás? Véleményem szerint igen. De hogyan lehetne bizonyítani? A segítségeteket előre is köszönöm. Laci

[758] s.addam2007-05-29 17:11:17

Hogy lehet legkevesebb művelettel megállapítani, hogy egy kör (ismert a középpontja, és a sugara) és egy polygon (ismertek a pontjai) metszete üres halmaz-e?

[757] HoA2007-05-25 09:30:07

A kívánt bizonyítás egyszerű az izoperimetrikus tétel felhasználásával: Adott kerületű síkidomok közül a kör zárja be a legnagyobb területet. Állítsuk csuklós szerkezetünket húrsokszög alakúra. Rajzoljuk meg a körülírt kört. Az ábrán zölddel jelölt holdacskákat ragasszuk hozzá az oldalakhoz. A kör területe = húrsokszög területe + holdacskák területe. Ezután mozdítsuk el a csuklós szerkezetet. A keletkező idom körívekből álló kerülete megegyezik az eredeti kör kerülettel, de területe kisebb a körénél. Az idom területe = nem-húrsokszög területe + holdacskák területe. A holdacskákat levonva : nem-húrsokszög területe < húrsokszög területe. QED.

Előzmény: [756] BohnerGéza, 2007-05-24 19:45:22
[756] BohnerGéza2007-05-24 19:45:22

Fordított utat jártam be, mint HoA. Azt szeretném bizonyítani, hogy egy csuklós n-szög által bezárt terület, azaz ha egy n-szög oldalai adottak, területe akkor maximális, ha húrsokszög.

Ha csuklós n-szögünket két csúszós síklap közé rakjuk és belül fölfújuk, maximális tárfogatra, azaz alapterületre törekszik. A 114** feladatban leírt erők a nyomás miatt kialakuló a erőhatásokat helyettesítik. A 114** bizonyítása a fenti tétel "fizikai" bizonyítása. ( Erre egy, a matematikában sokszor használt, középiskolában tanult, bizonyítási módszert ajánlok. )

Előzmény: [753] HoA, 2007-05-24 11:39:17
[755] lorantfy2007-05-24 15:31:32

Kedves Cogito!

Kösz a figyelmességet és az ellenpéldát! A többi hozzászólónak is köszönet, hogy kijavítottátok ezt a hibásan megfogalmazott állítást!

Előzmény: [749] Cogito, 2007-05-23 17:09:36
[754] sakkmath2007-05-24 14:52:17

Egy idevágó részlet egy nemrégi dolgozatomból:

Előzmény: [752] HoA, 2007-05-24 11:24:52
[753] HoA2007-05-24 11:39:17

114** feladat: Igen. Geomatriai bizonyítást egyelőre nem adok, hátha más is foglalkozik a feladattal. Egy "fizikai" bizonyítás: Tegyük fel, hogy kis csuklós szerkezetünket rá tudjuk fektetni egy síkban kifeszített szappanhártyára melyet n-szögünkön belül kiszúrunk. Ekkor a hártya a rudakra a feladat feltételeiben leírt erőhatásokat fejti ki. A szappanhártya minimális felületűre akar összehúzódni. Ha tudjuk, hogy egy csuklós n-szög által bezárt terület akkor maximális ha húrsokszög, adódik, hogy a hártya csuklós szerkezetünket húrsokszöggé alakítja és ez az állapot stabil.

Előzmény: [747] BohnerGéza, 2007-05-23 00:28:23
[752] HoA2007-05-24 11:24:52

Az O1 középpontú, \lambda1 nagyítású KH1 és O2 középpontú \lambda2 nagyítású KH2 középpontos hasonlóságok KH3 szorzatának O3 középpontját éppen annak alapján könnyű meghatározni, hogy O3 a fixpont. KH1O3-t P-be viszi, ahol P=O1+\lambda1.(O3-O1) . KH2 P-t Q-ba viszi, ahol Q=O2+\lambda2.(P-O2) = (1-\lambda2).O2+\lambda2.(O1+\lambda1.(O3-O1)) . Mivel O3 fixpont, Q=O3

(1-\lambda2).O2+\lambda2.(1-\lambda1)O1+\lambda1\lambda2O3=O3

 O_3 = \frac {(1-\lambda_2) \cdot O_2 + \lambda_2 \cdot ( 1 - \lambda_1 ) O_1}{1 - \lambda_1 \lambda_2}

O1ésO2 együtthatóinak összege 1, így O3 az O1O2 egyenesen van.

Előzmény: [751] Yegreg, 2007-05-23 22:34:29
[751] Yegreg2007-05-23 22:34:29

Maga az az állítás, hogy középpontos hasonlóságok szorzata középpontos hasonlóság, igaz, ha \prod\lambda_i\neq 1 (ha 1 a szorzat, akkor eltolás), méghozzá tényleg \prod\lambda_i arányú, a középpontja viszont nem az, ami le volt írva. Két hasonlóság esetén a szorzat középpontja a középpontok egyenesén van, méghozzá valahogy úgy, hogy \lambda_1O_1O_3=O_1O_2\pm\frac1{\lambda_2}O_2O_3, ahol \lambda-k az arányok, O1, O2 a középpontok, O3 a szorzat középpontja. Gondolom, O3-at meg lehet adni osztóviszonyokkal is, de már késő van hozzá, hogy kitaláljam :oS Egyébként szerintem ilyenek benne vannak a Geometria és határterületeiben, de nem biztos, majd megnézem.

[750] Lóczi Lajos2007-05-23 17:20:09

Vannak még figyelmes olvasók, legyen szó akár 3 év messzeségéről is. (Micsoda véletlen egybeesés: pár perce kaptam egy levelet, hogy egy általam 3 éve jelentett szoftverproblémát kijavítottak :-)

Előzmény: [749] Cogito, 2007-05-23 17:09:36
[749] Cogito2007-05-23 17:09:36

Tisztelt lorantfy!

Idézet a [151]-es hozzászólásodból:

Az állítás általánosan is megfogalmazható: Legyen K1 O középpontú \lambda1 arányú kph. K2 pedig P középpontú \lambda2 arányú kph. Ezek egymásutáni végrehajtása helyettesíthető egy K kph-al melynek aránya \lambda=\lambda1 \lambda2 középpontja pedig az a Q pont, mely P-nek O-ra vonatkozó \lambda arányú kph. képe.

E sorokat olvasva elgondolkoztam: ha igaz ez a tétel, vajon miért nem szerepel (például) Reiman István: A geometria határterületei című könyvében, illetve más szakkönyvekben? Hosszas tűnődések után fölmerült bennem a gyanú, hogy talán azért nem említik, mert esetleg hamis ez az általánosítás?!. Ilyenkor egy dolgot tehet az ember, próbál egy ellenpéldát keresni. Némi kutakodás után bukkantam a 'tétel' következő cáfolatára:

Legyen K1 az O középpontú, \lambda1=2 arányú középpontos hasonlóság, K2 pedig a P középpontú, \lambda2=2 arányú középpontos hasonlóság a következő választással: a számegyenesen az O pont legyen az origó, P legyen a 3-as pont.

Az OP szakasz O-hoz közelebbi H harmadolópontja az első nagyításnál a második harmadolópontba, H'-be kerül. Alkalmazzuk H'-re a P középpontú, \lambda2=2 arányú nyújtást. Ekkor H' képe H lesz. H tehát fixpont, ez a pont a szorzathasonlóság középpontja. A vitatott 'tétel' e konkrét esetre így szólna: P-nek O-ra vonatkozó, \lambda=4 arányú középpontos hasonlósági képe H. Ellentmondásra jutottunk, hiszen 1\ne12. A 'tétel' tehát hamis.

Ezek után kijelenthetjük: az általánosítás ilyen módon nem lehetséges (valószínűleg más módszerrel sem...). Üdvözlettel: Cogito

Előzmény: [151] lorantfy, 2004-07-16 00:54:41
[748] Iván882007-05-23 13:13:57

Ok. Akkor pontosítok. Valós függvényre gondoltam f:R\toR. Időközben rájöttem arra, hogy a sejtésem így nem igaz.

Ha az egyik aszimptota párhuzamos az y tengellyel, akkor a másik tetszőleges szöget zárhat be vele.(a ]0°;180°[ nyílt intervallumban). De mi a helyzet akkor, ha nem párhuzamos az y-tengellyel egyik aszimptota sem?

Szerintem akkor nem lehet fv. a grafikon képe, de ezt se bizonyítani se cáfolni nem tudom.

Előzmény: [745] Lóczi Lajos, 2007-05-22 10:58:49
[747] BohnerGéza2007-05-23 00:28:23

Másodszor módositott 114. feladat: Örülök, hogy HoA nem foglalkozott a feladattal. Ellenpéldája jó lett volna.

A módositott, legyen 114** feladat: Van egy n rúdból álló, a szomszédos rudakat csuklóval összekötött tömegtelen n-szögünk. Igaz-e, hogy ez n db olyan erő hatására, melyek a rudak felezőpontjában hatnak, a rudak hosszával arányosak és merőlegesek a rudakra, csakkor lehet nyugalomban, ha rúdhúrszöget alkotnak a rudak?

Nem triviális!!!

Előzmény: [746] BohnerGéza, 2007-05-22 16:27:35
[746] BohnerGéza2007-05-22 16:27:35

módositott 114. feladat: Örülök, hogy HoA foglalkozott a feladattal. Ellenpéldája jó.

A módositott, legyen 114* feladat: Van egy n rúdból álló, a szomszédos rudakat csuklóval összekötött tömegtelen n-szögünk. Igaz-e, hogy ez n db olyan erő hatására, melyek a rudak felezőpontjában hatnak és merőlegesek a rudakra, csakkor lehet nyugalomban, ha rúdhúrszöget alkotnak a rudak, az erők a rudak hosszával arányosak és erővonalaik átmennek a rúdsokszög köré írt kör középpontján?

Előzmény: [742] HoA, 2007-05-21 12:19:34
[745] Lóczi Lajos2007-05-22 10:58:49

Esetleg egy elforgatott síkbeli koordinátarendszerben!

Amúgy pl. egy zárt síkbeli körvonal is nyilván felfogható függvényként, legfeljebb nem valós-valós függvényként, hanem pl. R\toR2 függvényként, tehát a feladat megfogalmazása meglehetősen laza: meg kellene mondani, milyen halmazról melyikbe képezzen a függvényként felfogandó hozzárendelés.

Előzmény: [744] Sirpi, 2007-05-22 10:44:19
[744] Sirpi2007-05-22 10:44:19

Nem inkább az kellene, hogy az egyik aszimptota függőleges?

Előzmény: [743] Iván88, 2007-05-22 10:37:27
[743] Iván882007-05-22 10:37:27

115.feladat

Igaz-e, hogy egy hiperbola grafikonja csak akkor lehet függvény is, ha az aszimptotái merőlegesek egymásra?

[742] HoA2007-05-21 12:19:34

114. feladat: Nem. Ellenpélda az ábrán. ( képzeljük, hogy szimmetrikus :-)

Előzmény: [739] BohnerGéza, 2007-05-19 19:23:10
[741] jonas2007-05-21 10:16:41

Aha.

Akkor pedig zárt konvex halmazból is csak kontinuum sok van.

Előzmény: [740] Fálesz Mihály, 2007-05-20 15:01:24
[740] Fálesz Mihály2007-05-20 15:01:24

Egy m2 térben legfeljebb kontinuum sok nyílt halmaz lehet.

Veszel egy megszámlálható bázist, jelen esetben pl. az összes olyan nyílt téglalapot, aminek mind a 4 koordinátája racionális. Minden nyílt halmazhoz hozzárendeled az általa tartalmazott báziselemek (téglalapok) halmazát...

Zárt halmazból pedig ugyanannyi van, mint nyílt halmazból...

Előzmény: [734] jonas, 2007-05-17 15:52:16
[739] BohnerGéza2007-05-19 19:23:10

114. feladat: Talán fizika feladat? Van egy n rúdból álló, a szomszédos rudakat csuklóval összekötött tömegtelen n-szögünk. Igaz-e, hogy ez n db erő hatására csakkor lehet nyugalomban, ha rúdhúrszöget alkotnak a rudak, az erők a rudak felezőpontjában hatnak, a rudak hosszával arányosak és erővonalaik átmennek a rúdsokszög köré írt kör középpontján?

[738] Csimby2007-05-18 01:48:36

Ezek szerint rosz a megoldásom ;-)

Előzmény: [735] BohnerGéza, 2007-05-17 17:48:31
[737] Csimby2007-05-18 01:46:16

Tök jó kérdés, de már muszáj lefeküdnöm :-)

Előzmény: [734] jonas, 2007-05-17 15:52:16

  [1]    [2]    [3]    [4]    [5]    [6]    [7]    [8]    [9]    [10]